Final Review LCHS 2013.

215
Final Review LCHS 2013

description

Which type of rock listed below covers the majority of the Earth’s land surface? I. Igneous II. Metamorphic III. Sedimentary A. I only B. II only C. III only D. I and II E. II and III

Transcript of Final Review LCHS 2013.

Page 1: Final Review LCHS 2013.

Final ReviewLCHS 2013

Page 2: Final Review LCHS 2013.

Which type of rock listed below covers the majority of the Earth’s land surface?

I. IgneousII. MetamorphicIII. Sedimentary

A. I onlyB. II onlyC. III onlyD. I and IIE. II and III

Page 3: Final Review LCHS 2013.

CIgneous rocks are formed when magma (molten rock

deep within the Earth) cools and hardens. Sometimes the magma cools inside the Earth, and other times it erupts onto the surface from volcanoes as lava. Examples of this rock type include basalt and obsidian.

 Metamorphic rocks are formed under the surface of

the Earth from the change that occurs due to intense heat and pressure. The rocks that result from these processes often have ribbon-like layers and may have shiny crystals formed by minerals growing slowly over time on their surface. Examples of this rock type include gneiss and marble.

Page 4: Final Review LCHS 2013.

Which of the following statements is NOT true regarding clumped distribution patterns?A. It is the most common type of dispersion found in nature.B. The distance between neighboring individuals is maximized.C. It is found in environments that are characterized by patch resources.D. Organisms that usually serve as prey form clumped distributions in areas where they can hide.E. It can serve as a mechanism against predation as well as an efficient mechanism to trap or corner prey.

Page 5: Final Review LCHS 2013.

B: Clumped distribution is the most common type of dispersion found in nature. In clumped distribution, the distance between neighboring individuals is minimized. This type of distribution is found in environments that are characterized by patchy resources. Clumped distribution is the most common type of dispersion found in nature because animals need certain resources to survive, and when these resources become rare during certain parts of the year, animals tend to "clump" together around these crucial resources. Individuals might be clustered together in an area due to social factors such as herding and family groups. Organisms that usually serve as prey form clumped distributions in areas where they can hide and detect predators easily.

Page 6: Final Review LCHS 2013.

Canada's net population growth was 0.9% in 2006. What was its approximate doubling time?A. 9 yearsB. 19 yearsC. 42 yearsD. 58 yearsE. 78 years

Page 7: Final Review LCHS 2013.

EThe population doubling time is the period of time required

for a country to double in the number of people. When the relative growth rate is constant, the quantity undergoes exponential growth and has a constant doubling time or period that can be calculated directly from the growth rate. This time can be approximated by dividing 70 (more accurately by 72) by the percentage growth rate.

Doubling time = 70/0.9 = 78 years

Thus if the growth rate remains constant, Canada's population would double from its 2006 population of 33 million to 66 million by the year 2084

Page 8: Final Review LCHS 2013.

Which country listed below ranks largest in their percent of global agricultural output?A. United StatesB. ChinaC. IndiaD. RussiaE. European Union

Page 9: Final Review LCHS 2013.

BAgriculture is a vital industry in China, employing over

300 million farmers. China ranks first in worldwide farm output, primarily producing rice, wheat, potatoes, sorghum, peanuts, tea, millet, barley, cotton, oilseed, pork, and fish. Beginning in about 7700 BCE with millet agriculture, China's development of farming over the course of its history has played a key role in supporting the growth of what is now the largest population in the world.

 Although China's agricultural output is the largest in the

world, only about 15% of its total land area can be cultivated. China's arable land, which represents 10% of the total arable land in the world, supports over 20% of the world's population. Only about 1.2% of all arable land in China permanently supports crops.

Page 10: Final Review LCHS 2013.

Which of the following is most likely to occur in a forested region that has been recently clear-cut?(A) The concentration of nitrates in streams

running through the region will increase.(B) The average depth of topsoil will increase.(C) The water temperature in streams running

through the region will decrease.(D) Volume of runoff after rains will decrease.(E) The frequency of landslides will decrease.

Page 11: Final Review LCHS 2013.

Which specific air pollutant do CAFE standards NOT address?

A. Carbon monoxideB. Carbon dioxideC. NOx

D. SOx

E. Formaldehyde

Page 12: Final Review LCHS 2013.

B – First enacted by Congress in 1975, the purpose of Corporate Average Fuel

Economy (CAFE) is to reduce energy consumption by increasing the fuel economy of cars and light trucks.

The National Highway Traffic Safety Administration (NHTSA) sets fuel economy standards for cars and light trucks sold in the United States and administers the CAFE program, while the Environmental Protection Agency (EPA) provides the fuel economy data and manages emissions standards although state and local governments may enact stricter regulations. Currently there are three sets, or "tiers," of emission standards for light-duty vehicles in the United States. These standards specifically restrict emissions of carbon monoxide (CO), oxides of nitrogen (NOx), particulate matter (PM), formaldehyde (HCHO), non-methane organic gases (NMOG), and non-methane hydrocarbons (NMHC) and the amount of sulfur allowed in gasoline and diesel fuel, since sulfur can interfere with the operation of advanced exhaust treatment systems such as selective catalytic converters and particulate filters. Federal emission regulations addressed through CAFE standards do not cover the primary component of vehicle exhaust, carbon dioxide (CO2) since CO2 emissions are proportional to the amount of fuel used.

Page 13: Final Review LCHS 2013.

A senior-citizen housing development agency in a large, metropolitan area decided to incorporate the latest hurricane-resistant design features into a new housing project even though the area was not prone to frequent hurricanes. This is an example of

A. cost overrunB. mitigationC. remediationD. preservationE. restoration

Page 14: Final Review LCHS 2013.

B - Mitigation is the elimination or reduction of the frequency, magnitude, or severity of exposure to risks, or minimization of the potential impact of a threat or warning. Preservation includes activities designed to perpetuate existing facilities in their original condition. Restoration involves activities designed to repair existing facilities. Remediation involves activities in correcting a fault or deficiency in an existing facility.

Page 15: Final Review LCHS 2013.

The climatogram would be typical for aA. coniferous forestB. tropical rainforestC. grasslandD. desertE. taiga

Page 16: Final Review LCHS 2013.

A - Between the tundra to the north and the deciduous forest to the south lies the large area of coniferous forest. One type of coniferous forest, the northern boreal forest, is found between 50° to 60°N latitudes. Another type, the temperate coniferous forests, grows in lower latitudes of North America, Europe, and Asia, in the high elevations of mountains.

Coniferous forests consist mostly of conifers, trees that grow needles instead of leaves and cones instead of flowers. Conifers tend to be evergreen, that is, they bear needles all year long. These adaptations help conifers survive in areas that are very cold or dry. Some of the more common conifers are spruces, pines, and firs.

Precipitation in coniferous forests varies from 12" to 35" (300 to 900 mm) annually, with some temperate coniferous forests receiving up to 80" (2,000 mm). The amount of precipitation depends on the forest location. In the northern boreal forests, the winters are long, cold, and dry, whereas the short summers are moderately warm and moist. In the lower latitudes, precipitation is more evenly distributed throughout the year.

Page 17: Final Review LCHS 2013.

As the population of current developing countries undergo a demographic transition, which of the following are most likely to occur?

I. Urban populations will increase as people migrate to cities to find employment.II. Rural populations will increase as larger populations will require more food.III. Food shortages and famine will decrease as an increase in urban populations increases the efficiency of food distribution and supply.

A. I only D. I and IIIB. II only E. I, II, and IIIC. III only

Page 18: Final Review LCHS 2013.

B - Pollutants can be classified as primary or secondary. Primary pollutants are directly emitted from a process, such as ash from a volcanic eruption, carbon monoxide gas from a motor vehicle exhaust, or sulfur dioxide from factories. Secondary pollutants are not emitted directly. Rather, they form in the air when primary pollutants react or interact. An important example of a secondary pollutant is ground level (tropospheric) ozone—one of the many secondary pollutants that make up photochemical smog. Some pollutants may be both primary and secondary: that is, they are both emitted directly and formed from other primary pollutants.

Primary pollutants•Sulfur dioxide (SO2) is produced from burning coal mainly in thermal power plants. It causes smog and acid rain.•Nitrogen oxides (NOx) cause smog and acid rain. It is produced from burning fuels including petrol, diesel, and coal.•Carbon monoxide (CO) is a product of incomplete combustion of fuel such as natural gas, coal, or wood.•Chlorofluorocarbons (CFCs), released mainly from refrigeration, combine with a few other gases. This combination leads to a reduction of the ozone layer, which protects the Earth from the harmful ultraviolet rays of the sun.•Carbon dioxide (CO2) is a greenhouse gas emitted from combustion.•Particulate matter (PMx) includes smoke, dust, and vapor that can remain suspended for extended periods and can be introduced in the environment as primary as well as secondary pollutants.•Toxic metals, such as lead, cadmium, and copper, can enter to the environment through petrol, hair dye products, paints, batteries, etc.•Volatile organic compounds (VOC) include hydrocarbon fuel vapors and solvents.•Ammonia (NH3) is emitted from agricultural processes.•Odors emanate from garbage, sewage, and industrial processes.•Radioactive pollutants are produced by nuclear explosions and war explosives, as well as by natural processes such as radon. Secondary pollutants•Particulate matter is formed from gaseous primary pollutants and compounds in photochemical smog, such as nitrogen dioxide.•Ground level ozone (O3) is formed from NOx and VOCs.•Peroxyacetyl nitrate (PAN) is similarly formed from NO2 and VOCs.

Page 19: Final Review LCHS 2013.

A - The human population has lived a rural lifestyle through most of history. The world’s population, however, is quickly becoming urbanized as people migrate to the cities. In 1950, less than 30% of the world’s population lived in cities. The urban population grew to 47% in the year 2000, and it is expected to grow to over 60% by the year 2030.

Developed nations have a higher percentage of urban residents than less-developed countries. However, urbanization is occurring rapidly in many less-developed countries, and it is expected that most urban growth will occur in less-developed countries during the next decades.

The rapid growth of urban areas is the result of two factors: natural increase in population and migration to urban areas.

Migration is often explained in terms of either "push factors"—conditions in the place of origin that are perceived by migrants as detrimental to their well-being or economic security—and "pull factors"—the circumstances in new places that attract individuals to move there. Examples of push factors include high unemployment and political persecution, while examples of pull factors include increased job opportunities or greater educational opportunities.

Governmental policies also foster internal migration in many developing countries. For example, in order to pay foreign debt and to be more competitive in international markets, national governments have encouraged the export of national resources and agricultural products. Such products become natural resource capital that can be traded to bolster the national economies of developing countries. In order to produce agricultural products quickly, efficiently, and at lower prices, national governments often look to decrease the number of small producers and turn agricultural production and resource extraction over to larger enterprises, with larger production facilities and a lower per-unit cost of production. These policies turn land into a commodity that can be bought and sold, and it is viewed only in terms of its productive capabilities.

Page 20: Final Review LCHS 2013.

In 1995 a certain model of car had a rating of 20.0 miles per gallon. In 2010, the same model had a rating of 30.0 miles per gallon. What was the percentage change in car fuel mileage?

A. 25%B. 33%C. 50%D. 66%E. 133%

Page 21: Final Review LCHS 2013.

C – The general equation for percentage change is

(Nfinal – N initial) × 100 = % change N initial

(30 – 20) × 100 = 50.0% 20

Page 22: Final Review LCHS 2013.

Which of the following air pollutants is classified as a "secondary" air pollutant?

A. SOxB. Tropospheric ozoneC. NOxD. Stratospheric ozoneE. VOC

Page 23: Final Review LCHS 2013.

Zero population growth is associated with(A) phase I only(B) phase II only(C) phase III only(D) phase IV only(E) phases I and IV

Page 24: Final Review LCHS 2013.

The rate of population growth starts to slow down at which point?(A) The end of phase I(B) The middle of phase II(C) The beginning of phase III(D) The end of phase III(E) The middle of phase IV

Page 25: Final Review LCHS 2013.

Which of the following is most likely the primary cause of high death rates in phase I?(A) Loss of breeding-age males due to warfare(B) Loss of breeding-age females due to disease(C) Large percentage of elderly individuals in the population(D) Infant and childhood mortality(E) General starvation due to famine

Page 26: Final Review LCHS 2013.

Which of the following graphs shows human population growth over the last 1,000 years?A. B.

C. D. E.

Page 27: Final Review LCHS 2013.

A.

The human population has dramatically increased in recent times. Two thousand years ago, the Earth had approximately 300 million humans, about the population of the United States today. Two hundred years ago, the world population was about three times that number. Today, it is now more than 6 billion and growing rapidly. If current trends continue, there will be an additional 1 billion added to the world population every 13 or 14 years. 

This explosive growth came about because death rates fell faster than birth rates. The availability of antibiotics, immunizations, clean water, and increased food production yielded tremendous improvements in infant and child mortality. A rise in average life expectancy has also contributed to the surge in human numbers.

Fertility is a key factor in determining population growth over the long term. In order to maintain a stable population size, a total fertility rate (TFR) of 2.1 is needed and represents the total number of children an average woman will bear over her lifetime. The worldwide average TFR is 2.9, with the United States having a TFR of 2.1. Even with a TFR of 2.1, the United States and similar countries will still see its population increase, as children born in previous years reach adulthood and begin having offspring. Only after TFR values have remained low for a generation or longer will population size level off or decrease.

Page 28: Final Review LCHS 2013.

Approximately what percentage of the world’s fresh water is readily available for human use?

A. 0.00003B. 0.003C. 0.3D. 3E. 30

Page 29: Final Review LCHS 2013.

B. Approximately 97% of Earth’s water is found in the oceans, with only 3% of all water on Earth being fresh water. Of this fresh water, more than 68% is trapped and frozen in icecaps and glaciers, and about 1% is readily accessible in surface waters. Only 0.003% of Earth’s water is readily available and usable for human daily needs. To put this into perspective, if all the water on Earth could fit in a 26-gallon barrel, only 1 teaspoon of it would be fresh water readily available for human use.

Page 30: Final Review LCHS 2013.

The problem of bioaccumulation of pesticides was first brought to the public’s attention by

A. John MuirB. Aldo LeopoldC. Henry David ThoreauD. Rachel CarsonE. Garrett Hardin

Page 31: Final Review LCHS 2013.

D - Silent Spring written by Rachel Carson is widely credited with helping launch the environmental movement. When the book Silent Spring was published, Rachel Carson was already a well-known writer on natural history, but she had not previously been a social critic. The book was widely read and inspired widespread public concerns with pesticides and pollution of the environment. Silent Spring facilitated the ban of the pesticide DDT in 1972 in the United States. The book documented detrimental effects of pesticides on the environment, particularly on birds. Carson accused the chemical industry of spreading disinformation, and public officials of accepting industry claims uncritically.

Page 32: Final Review LCHS 2013.

Noise pollution can cause all of the following EXCEPT

A. stressB. increased respiration rateC. increased heart rateD. tinnitusE. aggression

Page 33: Final Review LCHS 2013.

B - Noise pollution is excessive, displeasing human, animal, or machine-created environmental noise that disrupts the activity or balance of human or animal life. High noise levels can contribute to cardiovascular effects in humans, a rise in blood pressure, an increase in stress and vasoconstriction, and an increased incidence of coronary artery disease. In animals, noise can increase the risk of death by altering predator or prey detection and avoidance, interfere with reproduction and navigation, and contribute to permanent hearing loss.

 Noise pollution affects both health and behavior. Unwanted sound (noise) can also damage physiological and psychological health. Noise pollution can cause annoyance and aggression, hypertension, high stress levels, tinnitus, hearing loss, sleep disturbances, and other harmful effects. Furthermore, stress and hypertension are the leading causes of health problems.

Chronic exposure to noise may cause noise-induced hearing loss. Older males exposed to significant occupational noise demonstrate significantly reduced hearing sensitivity than their non-exposed peers, though differences in hearing sensitivity decrease with time, and the two groups are indistinguishable by age 79.

High noise levels can contribute to cardiovascular effects, and exposure to moderately high levels during a single eight-hour period causes a statistical rise in blood pressure of five to ten points and an increase in stress and vasoconstriction leading to the increased blood pressure noted above as well as to increased incidence of coronary artery disease.

Noise can have a detrimental effect on animals, increasing the risk of death by changing the delicate balance in predator or prey detection and avoidance and interfering with the use of the sounds in communication especially in relation to reproduction and in navigation. Overexposure can lead to temporary or permanent loss of hearing.

An impact of noise on animal life is the reduction of usable habitat that noisy areas may cause, which in the case of endangered species may be part of the path to extinction. Noise pollution has caused the death of certain species of whales that beached themselves after being exposed to the loud sound of military sonar.

  

Page 34: Final Review LCHS 2013.

The Surface Mining Control and Reclamation Act of 1977I. Requires restoration of mined land to its former condition.II. Regulates the mining methods to be used.III. Grants free access to individuals to prospect for minerals on public land.A. I onlyB. II onlyC. III onlyD. II and IIIE. I, II, and III

Page 35: Final Review LCHS 2013.

A - The Surface Mining Control and Reclamation Act of 1977 (SMCRA) is the primary federal law that regulates the environmental effects of coal mining in the United States.

The regulation of active mines under SMCRA has several major components:Sets environmental standards that mines must follow while operating and achieve

when reclaiming mined land;Requires mining companies to obtain permits before conducting surface mining on

all land in the United States. Permit applications must describe what the pre-mining environmental conditions and land use are, what the proposed mining and reclamation will be, how the mine will meet the SMCRA performance standards, and how the land will be used after reclamation is complete;

Requires that mining companies post a bond sufficient to cover the cost of reclaiming the site;

Gives government regulators the authority to inspect mining operations and to punish companies that violate SMCRA or an equivalent state statute; and

Gives government regulators the authority to prohibit surface mining on certain lands, such as in National Parks and wilderness areas and allows citizens to challenge proposed surface mining operations on the ground that it will cause too much environmental harm

Page 36: Final Review LCHS 2013.

Exposure to small doses of a toxic chemical over a long period of time is known as

A. acute exposureB. chronic exposureC. hazardous exposureD. sustained exposureE. toxicity

Page 37: Final Review LCHS 2013.

B - Toxicity is the ability of a substance to produce an unwanted effect when the chemical has reached a sufficient concentration. The more toxic a material is, the smaller the amount of it necessary to be absorbed before harmful effects are caused (i.e., the lower the toxicity, the greater the quantity of it necessary to be absorbed}.

Hazardous is the probability that this concentration will occur while toxicity is an inherent property of the material. A material may be very toxic, but not hazardous, if it is handled properly and is not absorbed into the body. On the other hand, a material may have a very low toxicity but be very hazardous. Example: An open container of an acid is much more hazardous than a closed container of the same material. The toxicity of the material is the same, but the closed container is less hazardous as the probability of it causing harm is decreased.

Exposure can be classified as chronic or acute. In chronic exposures, the dose is delivered at some frequency over a period of time. In acute exposures, the dose is delivered in a single event, and absorption is rapid. Usually, a chronic exposure occurs at low concentration, and acute exposure at high concentration. Some materials may only cause harm if given acutely, not having any effect in the long term, whereas other materials may not exhibit an effect in the short term but may cause problems after prolonged exposure.

  

Page 38: Final Review LCHS 2013.

Approximately what percentage of all species of life that have ever lived on Earth are now extinct?

A. 0.1%B. 5%C.25%D. 50%E. 99%

Page 39: Final Review LCHS 2013.

E Most species of life that have existed on Earth have vanished as a result of normal or

background extinction, which fluctuates from 1 million years with mammals through 11 million years with some marine invertebrates. Besides normal, background extinction, animal life has experienced five mass extinctions, resulting in 50 to 95% of life disappearing within a limited time period. The first mass extinction occurred 440 million years ago, at the end of Ordovician, as a result of temperature fall and the resulting decrease in ocean levels. The second wave took place during the late Devonian, again due to temperature fall and ocean level lowering. During the third wave of extinction, at the end of Permian, approximately 250 million years ago, 95% of marine species and nearly 70% of terrestrial species disappeared due to active reconstruction of the Earth's crust and change of climate during the formation of the supercontinent Pangaea. The fourth extinction happened in the late Triassic, and the fifth and most recent one—resulting in the disappearance of dinosaurs—appeared about 65 million years ago. It was most likely due to the Earth colliding with an asteroid, resulting in cataclysmic tsunamis, acid rains, and the sea being covered by enormous amount of soil and organic matter, with only 12% of the then-existing species surviving on land.

The predicted sixth extinction will be anthropogenic in nature, or due to the activities of humans.

A recent analysis, published in the journal Nature, shows that it takes 10 million years before biological diversity even begins to approach what existed before a die-off. More than 10,000 scientists in the World Conservation Union have compiled data showing that currently 51% of known reptiles and insects and 73% of known flowering plants are in danger, as are many mammals, birds, and amphibians. It is likely that some species will become extinct before they are even discovered, before any medicinal use or other important features can be assessed. Given the current average extinction rate of 40 species a day, it would take only 16 thousand years for the extinction of 96% of all Earth’s current life forms.

Page 40: Final Review LCHS 2013.

Which of the following plant nutrients would most likely be a limiting factor?

A. NitrogenB. SulfurC. PhosphorusD. CalciumE. Compost

Page 41: Final Review LCHS 2013.

C – All plants have specific requirements for 17 basic nutrients. Of those 17 nutrients, 14 (including the primary nutrients of nitrogen, phosphorus, and potassium) are derived from the soil. Phosphorus is an essential nutrient and available for plants and animals in the form of the phosphate ion PO43−. It can be found in DNA molecules, molecules that store energy (ATP and ADP), and the fats of cell membranes. Phosphorus is also essential for certain parts of the human and animal body, such as the bones and teeth.

Phosphorus can be found on the Earth in water and soil. It is mainly cycling through water, soil, and sediments. Phosphorus moves slowly from deposits on land and in sediments, to living organisms, and much more slowly back into the soil and water sediment. The phosphorus cycle is the slowest one of the matter cycles.

Page 42: Final Review LCHS 2013.

Humans in developed countries would typically have what type of survivorship curve? 

A. IB. IIC. IIID.I and IIE. II and III

Page 43: Final Review LCHS 2013.

Examples of populations with Type I survivorship include humans in developed countries and animals in zoos. A lot of effort is invested in each individual, resulting in high survivorship throughout the life cycle and a state where most individuals die of old age. In general, this is more typical of K-selected species, which tend to grow in stable environments where intense competition between individuals is experienced. The heavy parental investment improves competitive ability and makes it more likely that individuals will survive to reproduction.

Page 44: Final Review LCHS 2013.

The city council of a small town was considering proposals from various special interest groups on what to do with a large tract of public land within its jurisdiction that contained a large number of trees. Listed below are several proposals from various groups. Which proposal listed below is most in line with the precautionary principle?

A. Do a study to determine the estimated sustainable yield of harvesting the trees. Then harvest trees below that level B. Do a study to determine the estimated sustainable yield of harvesting the trees. Then harvest trees at that levelC. Remove all trees and sell them for lumber, which would provide income to the city. Then build a parking lot on the site which would add a continuing source of revenue for the city.D. Remove all the trees and sell them for lumber. Then convert the land to a sustainable tree farm to make efficient and profitable in the future.D. Do nothing. Keep the land as it is.

Page 45: Final Review LCHS 2013.

A - The precautionary principle states that if an action or policy has a suspected risk of causing harm to the public or to the environment, in the absence of scientific consensus that the action or policy is harmful, the burden of proof that it is not harmful falls on those taking the action. This principle allows policy makers to make discretionary decisions in situations where there is the possibility of harm from taking a particular course or making a certain decision when extensive scientific knowledge on the matter is lacking. The principle implies that there is a social responsibility to protect the public from exposure to harm, when scientific investigation has found a plausible risk. These protections can be relaxed or modified only if further scientific findings emerge that provide sound evidence that no harm will result. The European Union has made the application of the precautionary principle a statutory requirement.

Areas typically concerned by the precautionary principle include•Global warming,•Extinction of species,•Introduction of new and potentially harmful products into the environment,

threatening biodiversity (e.g., genetically modified organisms),•Threats to public health, due to new diseases and techniques (e.g., AIDS transmitted

through blood transfusion),•Long-term effects of new technologies (e.g., health concerns regarding radiation

from cell phones),•Persistent or acute pollution (e.g., asbestos, endocrine disruptors),•Food safety,•Biosafety issues (e.g., recombinant DNA or "gene splicing").

Page 46: Final Review LCHS 2013.

An AP class was doing a field study of a lake. The class determined the amount of dissolved oxygen in the lake water at a depth of 2 feet through a titration technique and determined the amount to be 8 mg O2 per liter of lake water. Then the class filled and sealed two clear glass bottles with lake water from the same location and depth, labeling one bottle "Light" and the other bottle "Dark." The "Dark" bottle was wrapped completely with several layers of foil. Both bottles were then lowered 3 feet into the lake at the same location where the initial sample was taken. After one hour, the bottles were retrieved, and, again through titration, the amount of oxygen in the water in the "Light" bottle was determined to be 10 mg O2 per liter while the amount of oxygen in the water in the "Dark" bottle was determined to be 5 mg O2 per liter . The net primary productivity of the lake water sample wasA. 2 mg O2 per liter per hourB. 3 mg O2 per liter per hourC. 4 mg O2 per liter per hourD. 5 mg O2 per liter per hourE. 8 mg O2 per liter per hour

Page 47: Final Review LCHS 2013.

 A - Net primary productivity is just the energy going to new plant growth.  The water contains both microscopic plants and animals of the aquatic ecosystem. In the light bottle there is photosynthesis, or Gross Primary Production (GPP), and there is Respiration (R). The difference between these two processes is Net Primary Production: NPP = (GPP – R) NPP= (GPP – R)/Time = (10 mg O2/L - 8 mg O2/L )/1 hr = 2 mg O2/L /hr

Page 48: Final Review LCHS 2013.

An AP class was doing a field study of a lake. The class determined the amount of dissolved oxygen in the lake water at a depth of 2 feet through a titration technique and determined the amount to be 8 mg O2 per liter of lake water. Then the class filled and sealed two clear glass bottles with lake water from the same location and depth, labeling one bottle "Light" and the other bottle "Dark." The "Dark" bottle was wrapped completely with several layers of foil. Both bottles were then lowered 3 feet into the lake at the same location where the initial sample was taken. After one hour, the bottles were retrieved, and, again through titration, the amount of oxygen in the water in the "Light" bottle was determined to be 10 mg O2 per liter while the amount of oxygen in the water in the "Dark" bottle was determined to be 5 mg O2 per liter . What was the gross primary production (GPP)?A. 2 mg O2 per liter per hourB. 3 mg O2 per liter per hourC. 4 mg O2 per liter per hourD. 5 mg O2 per liter per hourE. 10 mg O2 per liter per hour

Page 49: Final Review LCHS 2013.

E - In the dark bottle, there is no photosynthesis, only respiration. Primary productivity is a term used to describe the rate at which plants and other photosynthetic organisms produce organic compounds in an ecosystem. There are two aspects of primary productivity: (1) net productivity, the organic materials that remain after photosynthetic organisms in the ecosystem have used some of these compounds for their cellular energy needs (cellular respiration), and (2) gross primary productivity, the entire photosynthetic production of organic compounds in an ecosystem. Since oxygen is one of the most easily measured products of both photosynthesis and respiration, a good way to gauge primary productivity in an aquatic ecosystem is to measure dissolved oxygen; however, gross productivity cannot be measured directly because respiration, which uses up oxygen and organic compounds, is always occurring simultaneously with photosynthesis.We can measure net productivity directly by measuring oxygen production in the light, when photosynthesis is occurring.We can also measure respiration without photosynthesis by measuring O2 consumption in the dark, when photosynthesis does not occur.Since Net Primary Productivity = Gross Primary Productivity – Respiration, we can calculate Gross Primary Productivity. GPP = (NPP + R)/ time = (2 mg O2/L + 8 mg O2/L)/1 hr = 10 mg O2/L /hr

Page 50: Final Review LCHS 2013.

Which of the following traits is NOT characteristic of an r-selected species?

A. low fertilityB. small body sizeC. early maturity onsetD. short gestation timeE. ability to disperse offspring widely

Page 51: Final Review LCHS 2013.

A - Examples of r-strategists include small rodents, insects, bacteria, and annual plants. r/K selection theory relates to the selection of combinations of traits in an organism that trade off between quantity and quality of offspring. The focus upon either increased quantity of offspring at the expense of individual parental investment or reduced quantity of offspring with a corresponding increased parental investment varies widely, seemingly to promote success in particular environments.Because of their higher reproductive rates and ecological opportunism, primary colonizers typically are r-strategists and are followed by a succession of increasingly competitive flora and fauna. Eventually a new equilibrium is approached (climax community) with r-strategists gradually being replaced by K-strategists, which are more competitive and better adapted to the emerging characteristics of the environment

Page 52: Final Review LCHS 2013.

Which of the following would most effectively slow the loss of arable land?(A)Increasing the efficiency of desalination processes(B)Reducing urban development(C)Increasing the efficiency of erosion control(D)Using more extensive irrigation systems(E)Discovering cheaper energy sources

Page 53: Final Review LCHS 2013.

Which of the following statements is NOT consistent with the second law of thermodynamics?I. When energy is converted from one form to another, the resulting form is always lower-quality energy and heat is generally given off.II. When energy is converted from one form to another, entropy increases; in other words, the energy is more dispersed.III. Energy can neither be created nor destroyed.A. I onlyB. II onlyC. III onlyD. I and IIIE. I, II and III

Page 54: Final Review LCHS 2013.

C - Thermodynamics is the study of energy. Energy is the ability to bring about change or to do work and exists in many forms (e.g., heat, light, chemical energy, and electrical energy).The First Law of Thermodynamics states that the total amount of energy and matter in the universe remains constant (it cannot be created nor destroyed), but that it can change from one form to another (e.g., electrical energy → heat energy).The Second Law of Thermodynamics is concerned with disorder or entropy. The universe is spontaneously moving in the direction of increased entropy or disorder. The driving force of all energy change therefore is the unstoppable tendency of energy to flow from high concentrations of energy (high quality) to lower concentrations of energy (low quality).Example: When a diesel engine turns a generator, diesel fuel is burned, which converts the concentrated chemical energy in the diesel fuel into heat energy through combustion and then into the engine's mechanical energy, which is then converted into electrical energy by the generator. However, not all of the energy released from the diesel fuel is converted to the final form—electricity. Some of the energy is lost through friction and heat; for example, the moving parts of the generator produce heat and the generator wires being heated up by internal friction as electrons flow through them, and the generator requires energy to run cooling fans to keep it cool. Finally, as the electrical energy flows through the electrical transmission lines, the wires are heated by the flowing electrons, and even more energy is lost as the electrical wires heat up the air surrounding them. All of this "other" energy is still there, but can no longer be used for the intended purpose of generating electricity.The Second Law of Thermodynamics also explains the progressive loss of energy as energy moves from a concentrated source (primary producer) to a higher order consumer through a food chain and explains why more people could be fed by eating plants and plant products than by eating meat.

Page 55: Final Review LCHS 2013.

The distribution of biomes around the world are in large part determined by long-term seasonal weather patterns which themselves are primarily determined by

A. the distance from the Earth to the sun during a particular time of the yearB. the amount of solar radiation that is released by the sun during certain periods of the yearC. the angle at which solar radiation reaches the EarthD. trade windsE. ocean currents

Page 56: Final Review LCHS 2013.

C - World biomes are primarily determined by climate. Climate is the long-term weather pattern of an area of the Earth, which includes the region's general pattern of weather conditions, seasons and weather extremes (e.g., hurricanes, droughts, or rainy periods). Two of the most important factors determining an area's climate are air temperature and precipitation.

The sun's rays hit the equator at a direct angle between 23° N and 23° S latitude. Radiation that reaches the atmosphere within these areas is at its most intense resulting in areas of the Earth that are the warmest. In all other cases, the rays arrive at an angle to the surface and are less intense. The closer an area is to the South or North Pole, the smaller the angle and therefore the less intense the radiation.

Page 57: Final Review LCHS 2013.

The practice of harvesting trees in a way that moves a forest stand toward an uneven-aged or all-aged condition or "structure" is known as

A. clear cuttingB. shelterwood cuttingC. old growth cuttingD. seed tree harvestingE. selective cutting

Page 58: Final Review LCHS 2013.

E - Silviculture is the practice of controlling the establishment, growth, composition, health, and quality of forests to meet diverse needs and values. Selection cutting is the practice of harvesting trees in a way that moves a forest stand toward an all-aged condition also known as "uneven-aged" structure.

"Selection" cutting is often confused with "selective" cutting, a term synonymous with the practice of high-grading (the removal of the best trees in a forest) often with a disregard for the future of the residual stand.

Page 59: Final Review LCHS 2013.

The most common isotope used as a fuel in nuclear reactors that produce electrical energy is

A. U235

B. U236

C. U238

D. Pu239

E. He4

Page 60: Final Review LCHS 2013.

Uranium-235 (U235) is an isotope of uranium making up about 0.72% of naturally occurring uranium. Before it can be used, however, its concentration must be increased or "enriched." Unlike the more predominant isotope uranium-238, U235 can sustain a chain reaction. It is the only fissile isotope that is found in significant quantity in nature, and it has a half-life of 700 million years.

During a chain reaction, a neutron is absorbed by a uranium-235 nucleus, turning it briefly into an excited uranium-236 nucleus, with the excitation energy provided by the kinetic energy of the neutron plus the forces that bind the neutron. The uranium-236, in turn, splits into fast-moving lighter elements (fission products) and releases three free neutrons and gamma radiation. The heat that is produced in this reaction is used by most nuclear power plants today to produce steam, which spins turbines connected to electrical generators.

Page 61: Final Review LCHS 2013.

The pollutant found in the largest amount in agricultural runoff is

A. soilB. animal wastesC. fertilizerD. pesticidesE. crop residue

Page 62: Final Review LCHS 2013.

A - Agriculture is a highly intensified industry in many parts of the world, producing a range of wastewaters requiring a variety of treatment technologies and management practices.

Soil washed off fields is the largest source of agricultural pollution. Excess sediment causes high levels of turbidity in water bodies, which can inhibit growth of aquatic plants, clog fish gills and smother animal larvae.

Farmers may utilize erosion controls to reduce runoff flows and retain soil on their fields. Common techniques include using contour plowing, mulching crops, rotating crops, planting perennial crops, and installing riparian buffers.

Nitrogen and phosphorus are key pollutants found in runoff, and they are applied to farmland as commercial fertilizer, animal manure, and municipal or industrial wastewater or sludge. These chemicals may also enter runoff from crop residues, irrigation water, wildlife, and atmospheric deposition

Page 63: Final Review LCHS 2013.

A country has a birth rate of 15.5 live births per 1,000 population and a death rate of 7.5 per 1,000 population. The population growth rate is

A. - 0.80B. - 0.08C. 0.00D. 0.08E. 0.80

Page 64: Final Review LCHS 2013.

E Growth Rate = (Birth Rate – Death Rate)/10 = (15.5 – 7.5)/10 = 0.8 Population growth is the change in a population over time and

can be quantified as the change in the number of individuals in a population per unit of time. Globally, the growth rate of the human population has been declining since peaking in 1962 at 2.20% per annum.

In 2009, the estimated annual growth rate was 1.1%. The world annual birth rate is 1.915%, the mortality rate is 0.812%, and the growth rate is 1.092%. The last one hundred years have seen a rapid increase in population due to medical advances and a massive increase in agricultural productivity made possible by the Green Revolution.

Page 65: Final Review LCHS 2013.

Which biome listed below would contain soil that is acidic and contains few nutrients?

A. DesertB. GrasslandC. Coniferous ForestD. Deciduous ForestE. Tropical Rainforest

Page 66: Final Review LCHS 2013.

E - Despite the rapid growth of vegetation in a tropical rainforest, soil quality is often quite poor. Rapid bacterial decay prevents the accumulation of humus. Most trees have roots near the surface because there are insufficient nutrients below the surface as most of the trees' minerals come from the top layer of decomposing leaves and animals. If rainforest trees are cleared, rain can accumulate on the exposed soil surfaces, creating run-off and beginning the process of soil erosion.

Page 67: Final Review LCHS 2013.

Most of the world’s garbage belongs to what category?

A. paperB. plasticC. organicD. metalE. glass

Page 68: Final Review LCHS 2013.

C - This year, the world will generate 2.6 trillion pounds of garbage. Almost half of it is classified as "organic," which consists of food, food animals eat, horticultural waste, green and garden waste, human and animal waste, bio-solids, and sludge.

Page 69: Final Review LCHS 2013.

Formation of photochemical smog does NOT depend upon the presence of

A. sunlightB. volatile organic compounds (VOCs)C. NOx

D. peroxyacyl nitrates (PANs)E. water vapor

Page 70: Final Review LCHS 2013.

D - PANs are produced in the formation of photochemical smog, they are not required for its formation.

Page 71: Final Review LCHS 2013.

Which of the following examples would NOT be an example of environmental remediation?A. Hauling contaminated soil to a regulated landfillB. Injecting surfactants into the subsurface to enhance desorption and recovery of pollutantsC. Pumping out groundwater and allowing the extracted groundwater to be purified by slowly proceeding through a series of vessels that contain materials designed to adsorb the contaminantsD. Injecting oxygen gas into a lake to promote growth of aerobic bacteria that will accelerate the breakdown of organic contaminants that have polluted the lakeE. Laying down sandbags and other barriers around a pond to keep possible runoff from a construction project from entering it

Page 72: Final Review LCHS 2013.

E - Environmental remediation involves the removal of pollution or contaminants for the general protection of human health and the environment. It consists of actions to minimize the effects of environmental damage that have already taken place (i.e., after the fact). Choice E is an example of environmental mitigation, which is the implementation of measures designed to reduce the undesirable effects of a proposed or future action that may affect the environment (i.e., before the damage occurs).

Page 73: Final Review LCHS 2013.

If an incandescent light bulb used for lighting has an efficiency rating of 5 percent, then for every 1.00 joule of electrical energy consumed by the bulb, which of the following is produced?(A) 1.05 joules of light energy(B) 1.05 joules of heat energy(C) 0.95 joule of light energy(D) 0.05 joule of light energy(E) 0.05 joule of heat energy

Page 74: Final Review LCHS 2013.

Stratospheric ozone depletion can cause all of the following EXCEPT

A. increase in skin cancerB. increased incidents of cataractsC. warming of the stratosphereD. reduction in crop yieldsE. increased rates of genetic mutations

Page 75: Final Review LCHS 2013.

C - The lower stratosphere appears to be cooling by about 0.5°C per decade. This cooling trend is interrupted by large volcanic eruptions, which lead to a temporary warming of the stratosphere and last for one to two years. Two reasons for stratospheric cooling are (1) depletion of stratospheric ozone and (2) increase in atmospheric carbon dioxide.

Less ozone leads to less absorption of ultraviolet radiation from the Sun. As a result, solar radiation is not converted into heat radiation in the stratosphere. So cooling due to ozone depletion is reduced as a consequence of reduced absorption of ultraviolet radiation. Ozone also acts as a greenhouse gas in the lower stratosphere. Less ozone means less absorption of infrared heat radiation and therefore less heat trapping.

Greenhouse gases (e.g., CO2, O3, and CFC) absorb infrared radiation from the surface of the Earth and trap the heat in the troposphere. If this absorption is strong, the greenhouse gas blocks most of the outgoing infrared radiation close to the Earth's surface. This means that only a small amount of outgoing infrared radiation reaches carbon dioxide in the upper troposphere and the lower stratosphere.

Carbon dioxide also emits heat radiation, which is lost from the stratosphere into space. In the stratosphere, this emission of heat becomes larger than the energy received from below by absorption. As a result, there is a net energy loss from the stratosphere, and cooling results.

Page 76: Final Review LCHS 2013.

The first step in the nitrogen cycle is when

A. nitrogen gas is absorbed by the plantsB. nitrogen gas is converted to ammoniaC. nitrites are converted to nitratesD. water reacts with nitratesE. ammonia breaks down to form nitrates

and nitrites

Page 77: Final Review LCHS 2013.

B. Nitrogen fixation is a process by which nitrogen (N2) in the atmosphere is converted into ammonia (NH3). Atmospheric nitrogen or elemental nitrogen (N2) is relatively inert: it does not easily react with other chemicals to form new compounds. Fixation processes free up the nitrogen atoms to be used in other ways. There are two main ways nitrogen is "fixed":

About 90% of nitrogen fixation is done by bacteria. Cyanobacteria convert nitrogen into ammonia and ammonium:

N2 + 3 H2 → 2 NH3 Ammonia can be used by plants directly.The energy from lightning causes nitrogen (N2) and water

(H2O) to combine to form ammonia (NH3) and nitrates (NO3−). Precipitation carries the ammonia and nitrates to the ground, where they can be assimilated by plants.

Nitrogen fixation is essential for all forms of life because nitrogen is required to biosynthesize basic building blocks of plants, animals and other life forms (e.g., nucleotides for DNA and RNA and amino acids for proteins). Therefore, nitrogen fixation is essential for agriculture and the manufacture of fertilizer.

Page 78: Final Review LCHS 2013.

Which of the following is NOT a common method employed in the cleanup of oil spills?A. skimmer boatsB. bacteriaC.oil dispersantsD. large floating boomsE. incineration

Page 79: Final Review LCHS 2013.

E. Oil is a flammable material. However, within a few hours at sea, it loses most of its volatile (flammable) components and picks up a high proportion of water. Consequently, burning the oil itself without first removing the water can be very difficult. Burning oily debris directly in the open is not a recommended method of disposal, except in very remote areas, due to the resultant smoke levels. When oil is burned by this method, it also tends to spread and to penetrate into the ground. Additionally, a tar-like residue may remain since it is rarely possible to achieve complete combustion.

However, a number of mechanisms are available for controlling oil spills and minimizing their impacts on human health and the environment. The key to effectively combating spills is careful selection and proper use of the equipment and materials best suited to the type of oil and the conditions at the spill site. Most spill response equipment and materials are greatly affected by such factors as conditions at sea, water currents, and wind. Damage to spill-contaminated shorelines and dangers to other threatened areas can be reduced by timely and proper use of containment and recovery equipment.

Mechanical containment or recovery is the primary line of defense against oil spills in the United States. Containment and recovery equipment includes a variety of booms, barriers, and skimmers, as well as natural and synthetic absorbing (sorbent) materials. Mechanical containment is used to capture and store the spilled oil until it can be disposed of properly.

Chemical and biological methods can be used in conjunction with mechanical means for containing and cleaning up oil spills. Dispersing agents and gelling agents are most useful in helping to keep oil from reaching shorelines and other sensitive habitats. Biological agents have the potential to assist recovery in sensitive areas such as shorelines, marshes, and wetlands. There are three kinds of oil-consuming bacteria: anaerobic sulfate-reducing bacteria, anaerobic acid-producing bacteria, and aerobic bacteria. These bacteria occur naturally and act to remove oil from an ecosystem.

Natural processes such as evaporation, oxidation, and biodegradation can start the oil spill cleanup process but are generally too slow to provide adequate environmental recovery. Physical methods, such as wiping with sorbent materials, pressure washing, and raking and bulldozing can be used to assist these natural processes.

Page 80: Final Review LCHS 2013.

What was the Earth’s original source of free atmospheric oxygen?

A. Oxygen produced from photosynthetic marine algaeB. Breakdown of stratospheric ozone by ultraviolet radiationC. Chemosynthesis by bacteriaD. Photosynthesis by cyanobacteriaE. Release of oxygen from deep sea floor vents and volcanoes

Page 81: Final Review LCHS 2013.

Answer: DThe earliest cells absorbed energy and food from the environment around them. They

used anaerobic fermentation, the breakdown of more complex compounds into less complex compounds and used the energy to grow and reproduce. The evolution of photosynthesis made it possible for cells to manufacture their own food.

Most of the life that covers the surface of the Earth depends directly or indirectly on photosynthesis. The most common form, aerobic photosynthesis, turns carbon dioxide, water, and sunlight into food. It captures the energy of sunlight in energy-rich molecules such as ATP, which then provide the energy to make sugars. To supply the electrons in the circuit, hydrogen is stripped from water, leaving oxygen as a waste product. Some organisms, including purple bacteria and green sulfur bacteria, use a form of anaerobic photosynthesis that use alternatives to hydrogen stripped from water as electron donors; examples are hydrogen sulfide, sulfur, and iron. Such organisms are mainly restricted to extreme environments such as hot springs and hydrothermal vents. These anaerobic forms of life arose about 3.8 billion years ago.

At first, the released oxygen was bound up with limestone, iron, and other minerals. The oxidized iron appears as red layers in geological strata called banded iron formations. When most of the exposed readily reacting minerals were oxidized, oxygen finally began to accumulate in the atmosphere. Though each cell only produced a minute amount of oxygen, the combined metabolism of many cells over a vast time transformed Earth’s atmosphere to its current state.

Photosynthesis had another major impact. Oxygen was toxic; much life on Earth probably died out as its levels rose in what is known as the oxygen catastrophe. Resistant forms survived and thrived, and some developed the ability to use oxygen to increase their metabolism and obtain more energy from the same food.

Page 82: Final Review LCHS 2013.

Which two greenhouse gases found in the Earth’s troposphere contribute most to the greenhouse effect? A. Carbon dioxide and water vaporB. Carbon dioxide and methaneC. Carbon dioxide and ozoneD. Methane and ozoneE. Water vapor and ozone

Page 83: Final Review LCHS 2013.

Answer: AA greenhouse gas is a gas in an atmosphere that absorbs and emits

radiation within the thermal infrared range. This process is the fundamental cause of the greenhouse effect. The primary greenhouse gases in the Earth's atmosphere are water vapor, carbon dioxide, methane, nitrous oxide, and ozone.

Since the beginning of the Industrial Revolution, the burning of fossil fuels has contributed to the increase in carbon dioxide in the atmosphere from 280 to 397 ppm, despite the uptake of a large portion of the emissions through various natural sinks involved in the carbon cycle. Anthropogenic carbon dioxide emissions come from combustion of carbon-based fuels, principally wood, coal, oil, and natural gas.

When these gases are ranked by their direct contribution to the greenhouse effect, the most important are Water vapor, which makes up 36–72% of the contribution percentage, and Carbon dioxide, which makes up 9–26% of the contribution percentage.

Page 84: Final Review LCHS 2013.

The primary source of fuel used in households in developing countries isA. oilB. woodC. crop residueD. animal dungE. grasses

Page 85: Final Review LCHS 2013.

Answer: BFor both the urban and rural poor in developing countries,

wood is usually the principal source of energy for cooking food and for keeping warm. In these countries, an estimated 86% of all the wood consumed annually is used as fuel. As populations have grown, this dependence has resulted in the destruction of the forest. Wood fuels account for two-thirds of all energy other than human and animal energy used in Africa, for nearly one-third consumed in Asia, for one-fifth in Latin America, and for 6% in the Near East. This compares with the one-third of 1% of total energy use, which wood fuels account for in developed countries.

About half of this wood fuel is used for cooking, about one-third is used for heating the house, boiling water, etc., and the remainder is used for other domestic purposes, for agricultural processing, and for industry.

 

Page 86: Final Review LCHS 2013.

Which layer of soil often contains aluminum, humus, iron, and clay leached from higher soil layers? A. AB. B C. CD. OE. E

Page 87: Final Review LCHS 2013.

Answer: BEluviation is the process by which clay is removed from

topsoil, and illuviation is the process by which clay is deposited in lower soil zones. Layer B, also known as subsoil, is the layer of soil under the topsoil on the surface of the ground. It is the zone of accumulation where rain water percolating through the soil has leached material from above and where it has precipitated and accumulated. Well-drained soils typically have the brightest colors within the B horizons. Subsoil may include substances such as sand, silt, and/or clay that have only been partially broken down by air, sunlight, water, wind, etc., to produce true soil. Below the subsoil is the substratum, which can be residual bedrock, sediments, or deposits, largely unaffected by soil-forming factors active in the subsoil. It contains partially weathered particles and some microorganisms. It is usually a lighter shade of brown or yellow. It contains the deeper roots of trees, but otherwise not much lives here.

Page 88: Final Review LCHS 2013.

Which Kingdom listed below contains the greatest number of species?A. Monera, which includes unicellular and colonial bacteria, including the true bacteria and blue-green algaeB. Protista, which includes unicellular protozoans and unicellular and multicellular algaeC. FungiD. PlantE. Animal

Page 89: Final Review LCHS 2013.

Answer: EThere are currently about 1.6 million species of living

organisms on Earth. This number may be much higher because new species are continually being discovered each day, particularly insects and nematodes in remote tropical regions. Of this 1.6 million, there are about 10,000 different species of bacteria, 250,000 different species of protists, 100,000 different species of fungi, 250,000 different species of plants, and about 1,000,000 different species of animals known at this time. More than half of all animal species are insects (800,000 species), and beetles (300,000 species) comprise the largest order of insects (one-fifth of all species on Earth).

Page 90: Final Review LCHS 2013.

Which groundwater contaminant listed below does NOT result from the mining of coal? I. Polychlorinated biphenyls (PCB) II. Acid III. Volatile organic compounds (VOC)A. I onlyB. II onlyC. III onlyD. I and IIIE. I, II, and III

Page 91: Final Review LCHS 2013.

Countries undergoing rapid population growth include which of the following?(A) I only(B) II only(C) III only(D) II and III only(E) I, II, and III

Page 92: Final Review LCHS 2013.

Approximately what percent of the population in Country II is under age 15?(A) 10 %(B) 15%(C) 20%(D) 25 %(E) 30%

Page 93: Final Review LCHS 2013.

Answer: DWhen pyrite rocks (rocks containing iron sulfide,

FeS2) associated with coal mining are exposed to oxygen, they react to generate acid mine drainage. The chemical reactions are similar to geologic weathering, which takes place over hundreds to thousands of years, but the rates of reaction are orders of magnitude greater than in "natural" weathering systems. The accelerated reaction rates can release damaging quantities of acidity, metals, and other soluble components into the environment, and they may ultimately seep into groundwater. The oxidation of pyrite, which occurs in high-sulfur coal mining operations, is among the most acid-producing of all weathering processes in nature.

Page 94: Final Review LCHS 2013.

Which of the following conditions would contain the most amount of water vapor in a given volume of air?A. 80oF at 10% relative humidityB. 10oF at 80% relative humidityC. 80oF at 80% relative humidityD. 10oF at 10% relative humidityE. 50oF at 80% relative humidity

Page 95: Final Review LCHS 2013.

Answer: CRelative humidity describes the amount of water

vapor in a mixture of air and water vapor. It is the ratio of the actual amount of water vapor in a given volume of air to the amount that could be present if the air were saturated at the same temperature and is commonly expressed as a percentage. With all other factors held constant, warmer air can hold more water vapor than cooler air. Generally, relative humidity doubles with each 20°F decrease, or halves with each 20°F increase in temperature; therefore, as temperature goes up, relative humidity goes down and vice versa.

Page 96: Final Review LCHS 2013.

Which two air pollutants listed directly cause the most deaths due to lung cancer in the United States?A. Tobacco smoke and asbestosB. Tobacco smoke and occupational exposureC. Radon gas and ozoneD. Carbon monoxide and smogE. Smog and ozone

Page 97: Final Review LCHS 2013.

Answer: BLung cancer is the leading cancer killer in both men and women in the

United States. In 1987, it surpassed breast cancer to become the leading cause of cancer deaths in women, and it causes more deaths than the next three most common cancers combined (colon, breast, and prostate).

 It has been estimated that active smoking is responsible for close to 90%

of lung cancer cases; occupational exposures to carcinogens account for approximately 9 to 15%, radon causes 10%, and outdoor air pollution 1 to 2%. Exposure to radon is a leading cause of lung cancer, accounting for an estimated 15,000 to 22,000 lung cancer deaths each year. Radon is a tasteless, colorless, and odorless gas produced by decaying uranium and occurs naturally in soil and rock. The majority of these deaths occur among smokers since there is a greater risk for lung cancer when smokers also are exposed to radon.

 Lung cancer can also be caused by occupational exposures, including

asbestos, uranium, and coke (an important fuel in the manufacture of iron in smelters, blast furnaces, and foundries), although these percentages are likely to decrease in the future due to awareness of the causes.

Page 98: Final Review LCHS 2013.

Almost one-third of the anthropogenic carbon dioxide emitted into the atmosphereA. is taken in by plants and converted to oxygen through photosynthesisB. is absorbed by the oceansC. naturally decomposes in the presence of ultraviolet radiationD. remains in the atmosphereE. combines with elements in the soil and is captured in a variety of “sinks”

Page 99: Final Review LCHS 2013.

Answer: BThe global oceans are the largest natural reservoir for carbon dioxide, absorbing

approximately one-third of the carbon dioxide added to the atmosphere by human activities each year. Over the next millennium, they are expected to absorb approximately 90% of the CO2 emitted to the atmosphere.

The Southern Ocean makes a substantial contribution to this oceanic carbon sink; more than 40% of the anthropogenic carbon dioxide in the ocean has entered south of 40° S. The rate-limiting step in the oceanic sequestration of anthropogenic carbon dioxide is the transfer of carbon across the base of the surface mixed layer into the ocean interior.

It is believed that dissolved CO2 in the ocean surface will double over its pre-industrial value by the middle of this century, with accompanying surface ocean acidity (pH) and carbonate ion (CO32-) decreases that are greater than those experienced during the transition from ice ages to warm ages. The uptake of anthropogenic CO2 by the ocean changes the chemistry of the oceans and can potentially have significant impacts on the biological systems in the upper oceans.

 Recent field and laboratory studies reveal that the carbonate chemistry of seawater has a profound negative impact on the calcification rates of individual species and communities in both planktonic (floating) and ocean bottom organisms. The effects of decreased calcification in microscopic algae and animals could impact marine food webs and, combined with other climatic changes in salinity, temperature, and upwelled nutrients, could substantially alter the biodiversity and productivity of the ocean.

Page 100: Final Review LCHS 2013.

Which factor listed below is NOT a factor in determining an ecological footprint? I. Population size II. Affluence III. Level of technologyA. I onlyB. II onlyC. III onlyD. II and III E. All factors are used in determining an ecological footprint

Page 101: Final Review LCHS 2013.

Answer: EThe ecological footprint is a measure of the human demand on the Earth's ecosystems. It is a

standardized measure of demand for natural capital that may be contrasted with the planet's capacity to regenerate. It represents the amount of biologically productive land and sea area necessary to supply the resources a human population consumes and to assimilate associated waste. Using this concept, it is possible to estimate how much of the Earth (or how many planet Earths) it would take to support humanity if everybody followed a given lifestyle. For 2007, humanity's total ecological footprint was estimated at 1.5 planet Earths; that is, humanity uses ecological services 1.5 times as quickly as Earth can renew them.

I = P × A × T is the formula that describe the impact of human activity on the environment, where human impact (I) on the environment equals the product of population (P) × affluence (A) × technology (T).

 Increased population (P) increases environmental impacts, which include but are not limited toincreased land use, which results in habitat loss for other species;increased resource use, which results in changes in land cover;increased pollution, which causes climate change, sickens people , and damages ecosystems. Affluence (A) represents the average consumption of each person in the population. As the

consumption of each person increases, the total environmental impact increases as well. A common tool used for measuring consumption is through GDP per capita. Even though GDP per capita measures production, it is often assumed that consumption increases when production increases and has been rising steadily over the last few centuries and is driving up human impact on the environment. 

Technology (T) represents how resource intensive the production of affluence is and how much environmental impact is involved in creating, transporting, and disposing of the goods, services, and amenities used. Improvements in efficiency can reduce resource intensiveness, reducing the effect of T. For example, an appropriate unit for T might be greenhouse gas emissions per unit of GDP.

Page 102: Final Review LCHS 2013.

Including the external costs into the price of a car is known as

A. optimum-cost pricingB. cost-benefit pricingC. added benefit pricingD. external-cost pricingE. full-cost pricing

Page 103: Final Review LCHS 2013.

Answer: EAn external cost occurs when producing or consuming a good or service imposes a cost upon a third party. If there are external costs in consuming a good (negative externalities), the social cost will be greater than the private cost. In the case of buying a car, driving a car imposes a private cost on the driver (the cost of fuel, insurance, maintenance, tax, and the car itself). However, driving a car creates costs to other people in society. These costs can include greater congestion and slower journey times for other drivers; the cause of death for pedestrians, cyclists, and other road users; and pollution and other health-related problems. Including these social costs into the cost of the car instead of paying for social costs through higher taxes is known as full-cost pricing.

Page 104: Final Review LCHS 2013.

A pipeline ruptured and released an industrial chemical into a lake. The initial concentration of the pollutant was measured at 50 ppm. If the pollutant dissipates by 20% every 7 days , what would the concentration of the pollutant in the lake water be after 14 days?A. 45 ppmB. 40 ppmC. 32 ppmD. 16 ppmE. 4 ppm

Page 105: Final Review LCHS 2013.

The pollutant is initially at a concentration of 50 ppm. After 7 days, 20% of the 50 ppm would have dissipated (0.50 × 20 ppm = 10 ppm); therefore, the concentration would be 50 original – 10 dissipated = 40 left. After another 7 days, 20% of the 40 ppm would have dissipated; therefore, the concentration would be 40 ppm – 8 ppm = 32 ppm.

Page 106: Final Review LCHS 2013.

Which action listed below would be the most effective in reducing the emissions of carbon dioxide gas in the United States?A. Switching from burning coal to produce electricity to using alternative methods of producing energy such as wind, solar, etc.B. Switching from gasoline and diesel engines to hybrids or battery-driven vehiclesC. Planting millions of treesD. Charging a large tax on the usage of electricity produced from burning coalE. Installing large air pumps to blow air through filters and ocean water to sequester and trap CO2

Page 107: Final Review LCHS 2013.

Answer: ACarbon dioxide (CO2) is the primary greenhouse gas emitted through human

activities. Carbon dioxide emissions in the United States increased by about 12% between 1990 and 2010 and accounts for about 84% of all U.S. greenhouse gas emissions from human activities, and CO2 emissions in the United States are projected to grow by about 1.5% between 2005 and 2020. The main human activity that emits CO2 is the combustion of fossil fuels (coal, natural gas, and oil) for energy and transportation, although certain industrial processes and land-use changes also emit CO2.

The combustion of fossil fuels to generate electricity is the largest single source of CO2 emissions in the United States, accounting for about 40% of total U.S. CO2 emissions and 33% of total U.S. greenhouse gas emissions. The type of fossil fuel used to generate electricity will emit different amounts of CO2. To produce a given amount of electricity, burning coal will produce more CO2 than will oil or natural gas.

The combustion of fossil fuels such as gasoline and diesel is the second largest source of CO2 emissions, accounting for about 31% of total U.S. CO2 emissions and 26% of total U.S. greenhouse gas emissions. This category includes transportation sources such as highway vehicles, air travel, marine transportation, and rail.

Various industrial processes account for about 14% of total U.S. CO2 emissions and 20% of total U.S. greenhouse gas emissions.

Planting trees alone, without reducing the burning of fossil fuels, does not solve the problem of CO2 emission, and many areas (i.e., arid areas) may not be suitable for tree planting. Charging a large tax on energy that generates CO2 gas disproportionately penalizes those on low or fixed incomes. Removing CO2 from the atmosphere and putting it into the oceans--- well, you know that one!

Page 108: Final Review LCHS 2013.

Most of the world’s earthquakes occurA. in the middle of continentsB. along the edges of continents or

tectonic platesC. in the open oceansD. on islandsE. during volcanic eruptions

Page 109: Final Review LCHS 2013.

Answer: BThe world's earthquakes are not randomly distributed over the Earth's surface but tend to be

concentrated in specific areas. The distribution of earthquakes is explained by the theory of plate tectonics, which combines many of the ideas about continental drift and sea-floor spreading. According to the theory of plate tectonics, the Earth's rigid outer shell (lithosphere) is broken into a mosaic of constantly moving oceanic and continental plates that slide over the uppermost layer of the mantle known as the asthenosphere. They interact along their margins, where one plate is forced under the other and where geological processes take place, such as the formation of mountain belts, earthquakes, and volcanoes.

Plate tectonics confirms that there are four types of seismic zones:The first follows the boundaries of mid-ocean ridges where the plates are pulled apart and

where the lithosphere is very thin and weak, so the strain cannot build up enough to cause large earthquakes. Earthquake activity here is low, and it occurs at very shallow depths. An example of this area would be Iceland.

The second type of earthquake is the shallow-focus event unaccompanied by volcanic activity. In these faults, two mature plates are scraping past each other. The friction between the plates can be so great that very large strains can build up before they are periodically relieved by large earthquakes. The San Andreas Fault is a good example of this.

The third type of earthquake is related to the collision of oceanic and continental plates. One plate is thrust or subducted under the other plate so that a deep ocean trench is produced. Ocean trenches are associated with curved volcanic island arcs on the landward plate. An example occurs near the Java trench.

The fourth type of seismic zone occurs along the boundaries of continental plates. Within this zone, shallow earthquakes are associated with high mountain ranges where intense compression is taking place. Intermediate- and deep-focus earthquakes also occur. Examples of these types of earthquakes occur in the Himalayan Mountains.

Page 110: Final Review LCHS 2013.

A sample of radioactive waste has a half-life of 10 years and an activity level of 2 curies. After how many years will the activity level of this sample be 0.25 curie?(A) 10 years(B) 20 years(C) 30 years(D) 40 years(E) 80 years

Page 111: Final Review LCHS 2013.

Which of the following is NOT an example of a point source of pollution?

A. Water pollution from an oil refinery wastewater discharge outlet

B. Noise pollution from a jet engineC. Agricultural runoff containing a high level

of nitrogenD. Light pollution from an intrusive street lightE. Thermal pollution from a nuclear power

plant 51

Page 112: Final Review LCHS 2013.

Answer: CPoint source pollution is related to emissions that can be easily identified as

coming from a single discharge source.Agricultural operations account for a large percentage of all nonpoint source

pollution in the United States. When large tracts of land are plowed to grow crops, it exposes and loosens soil that was once buried. This makes the exposed soil more vulnerable to erosion during rainstorms. It also can increase the amount of fertilizer and pesticides carried into nearby bodies of water.

To control sediment and runoff, farmers may utilize erosion controls to reduce runoff flows and retain soil on their fields. Common techniques include implementing contour plowing, crop mulching, crop rotation, perennial crops, and riparian buffers. Conservation tillage is a concept used to reduce runoff while planting a new crop. With this technique, the farmer leaves some crop reside from the previous planting in the ground to help prevent runoff during the planting process.

Nutrients (nitrogen and phosphorus) are typically applied to farmland as commercial fertilizer; animal manure; or spraying of municipal or industrial wastewater (effluent) or sludge. Nutrients may also enter runoff from crop residues, irrigation water, wildlife, and atmospheric deposition. Farmers can develop and implement nutrient management plans to reduce excess application of nutrients.

Page 113: Final Review LCHS 2013.

Which of the following statements would be TRUE regarding the following age structure diagrams?I. A would be typical for a developing country, while B would be typical for a developed country.II. A has a population that is declining, while B has a population that is expanding.III. A would be typical for a country like Japan, while B would be typical for a country like Mexico. A. I onlyB. II onlyC. III onlyD. II and IIIE. I and III

Page 114: Final Review LCHS 2013.

52 Answer: AA population pyramid, also called an age structure diagram, is a

graphical illustration that shows the distribution of various age groups in a population (typically that of a country or region of the world). When the population is growing, it forms the shape of a pyramid. It is also used to determine the overall age distribution of a population; an indication of the reproductive capabilities and likelihood of the continuation of a species.

It typically consists of two back-to-back bar graphs, with the population plotted on the X-axis and age on the Y-axis, one showing the number of males and one showing females.

The age structure diagram for Mexico, a developing country, shows the unmistakable pyramidal shape caused by ever-increasing number of births.

Japan’s diagram, which is typical for developed countries, has the classic shape of a shrinking population. Note how pre-reproductive age groups (0–14 years) have smaller populations than the reproductive age groups (15–44 years).

Page 115: Final Review LCHS 2013.

Mitigation of overgrazing includes all of the following techniques EXCEPT

A. using rotational grazing schedulesB. selecting proper breeds of cattle that work best with the natural ecology and resourcesC. planting warm-season perennial grasses such as switchgrassD. applying lime and organic fertilizer to the rangelandE. increasing the size of the herds to optimize profit

Page 116: Final Review LCHS 2013.

53 Answer: ESustainable pasture management practices, which include a balance of matching

forage and livestock resources, managing resources, proper breed selection, and looking for alternative feeds, can help to reduce the effects of overgrazing.

Properly managed grazing can have some benefits. Cattle manure fertilizes pastures. In addition, grazing can encourage re-growth and prevent the spread of noxious weeds. Ranchers have found that cattle hooves break up ground that left alone would be too hard for seeds to penetrate. Ranchers using this system have been able to double the carrying capacity of their pastures. Also they have a higher percentage of perennial grasses (which produce more biomass) as ground cover than land ranched conventionally.

Properly managed grazing maintains healthy vegetation, which helps to filter pollutants from runoff, reduce runoff velocity, and control soil erosion. Management practices that help to maintain vegetative cover involve distributing cattle so that they do not overgraze portions of pasture and allowing for recovery of the vegetation following a grazing period.

Adjusting the stocking rate seasonally, particularly in sensitive watershed areas, can also reduce the impacts of overgrazing.

Page 117: Final Review LCHS 2013.

Environmental consequences of hydraulic fracturing, also known as "fracking," include all of the following EXCEPT

A. blowouts and explosionsB. major spillsC. atmospheric emissions of toxic gasesD. land subsidenceE. decrease in cost of electricity

Page 118: Final Review LCHS 2013.

54 Answer: EHydraulic fracturing or "fracking" is the process

of utilizing pressurized water, chemical additives, and physical proppants (materials that will keep a fracture open) to fracture rock layers and release petroleum, natural gas, or other substances so that they can be extracted.

Environmental risks associated with this technology include contamination of groundwater, risks to air quality, the potential migration of gases and hydraulic fracturing chemicals to the surface, the potential mishandling of waste, and health effects.

Page 119: Final Review LCHS 2013.

In the food web, the owl is a I. Primary consumerII. Secondary consumerIII. Tertiary consumer

A. I onlyB. II onlyC. III onlyD. II and IIIE. I, II and III

Page 120: Final Review LCHS 2013.

55 Answer: DConsumers are organisms receive their energy by consuming

other organisms. These organisms are also referred to as heterotrophs, which include animals, bacteria and fungus. Such organisms may consume by various means, including predation, parasitization, and biodegradation.

Within an ecological food chain, consumers are categorized into three groups: primary consumers, secondary consumers, and tertiary consumers. Primary consumers are usually herbivores, feeding on plants and fungus (e.g. the squirrel or rabbit feeding only on plant material). Secondary consumers prey on the primary consumers (e.g. the owl feeding on the squirrel). Omnivores feed on both plants and animals (e.g. the bird feeding on either plants or insects). Tertiary consumers feed on secondary consumers (e.g. owl → mouse → insect → plants).

Page 121: Final Review LCHS 2013.

If the autotrophs or producers in the diagram received 1000 Kcal of solar energy, how much energy will be left for the owl if the owl consumed a mouse?A. 1 KcalB. 10 KcalC. 100 KcalD. 700 KcalE. 1000 Kcal

Page 122: Final Review LCHS 2013.

56 Answer: AGenerally, each trophic level relates to the one below it by absorbing and using

some of the energy it consumes for its own needs. When a food chain is diagrammed to illustrate the amount of energy that moves from one feeding level to the next, it is called an energy pyramid.

The efficiency with which energy or biomass is transferred from one trophic level to the next is called the ecological efficiency. Consumers at each level convert on average only about 10% of the chemical energy in their food to their own organic tissue. For this reason, food chains rarely extend for more than five or six levels. At the lowest trophic level (the bottom of the food chain or food pyramid), plants convert only about 1% of the sunlight they receive into chemical energy. Therefore, for every 1,000 Kcals of energy that is initially trapped in plants (autotrophs), only 100 Kcals of that energy will be ultimately available to the next higher level or primary consumers (i.e. insects); with 10% of that or 10 Kcals of energy being available to the mice; and only 1 Kcal of that energy ultimately being available for the owl. It follows then, that the owl needs to eat a lot of mice in order to staisfy its nutritional requirement, and that any particular environment can only support a limited number of tertiary consumers.

Page 123: Final Review LCHS 2013.

A 5-MW wind turbine can produce 15 million kWh of electrical energy in a year. The average U.S. household consumes about 10,000 kWh of electricity each year. If a company wishes to install 50 of these 5-MW wind turbines on a proposed wind farm, how many households can be supported in one year?

A. 5,000B. 7,500C. 25,000D. 50,000E. 75,000

Page 124: Final Review LCHS 2013.

57

Page 125: Final Review LCHS 2013.

Which of the following pollutants would NOT be classified as a persistent organic pollutant (POP)?

A. DioxinB. Polychlorinated biphenyl (PCBs)C. DDTD. ChlordaneE. Mercury

Page 126: Final Review LCHS 2013.

58 Answer: EPersistent organic pollutants (POPs) are organic compounds (contain carbon) that

are resistant to environmental degradation through chemical, biological, and photolytic processes. Because of this, they have been observed to persist in the environment, be capable of long-range transport, bioaccumulate in human and animal tissue, biomagnify in food chains, and have potential significant impacts on human health and the environment.

Many POPs are currently or were in the past used as pesticides. Others are used in industrial processes and in the production of a range of goods such as solvents, polyvinyl chloride, and pharmaceuticals. There are a few natural sources of POPs, but most POPs are created by humans in industrial processes.

POP exposure can cause death and illnesses including disruption of the endocrine, reproductive, and immune systems; neurobehavioral disorders; and cancers possibly including breast cancer. Exposure to POPs can take place through diet, environmental exposure, or accidents.

The element mercury (Hg), although toxic and a pollutant, is not organic as it does not contain the element carbon.

Page 127: Final Review LCHS 2013.

Which of the following actions would lead to an increase in biodiversity?I. Reducing or eliminating invasive speciesII. Introducing and maintaining invasive speciesIII. Decreasing the pH of acid rain 

A. I onlyB. II onlyC. III onlyD. I and IIIE. I, II, and III

Page 128: Final Review LCHS 2013.

59 Answer: ABetween 50 and 150 species become extinct daily, a level 10,000 times greater

than the natural rate of species extinction. The main drivers of biodiversity loss are deforestation, desertification, overexploitation, invasive species, pollution, and climate change. For a species to become invasive, it must successfully out-compete native organisms, spread through its new environment, increase in population density, and harm ecosystems in its introduced range.

Common characteristics of invasive species include rapid reproduction and growth, high dispersal ability, ability to adapt to new conditions, and ability to survive on various food types and in a wide range of environmental conditions. A good predictor of invasiveness is whether a species has successfully or unsuccessfully invaded elsewhere.

Ecosystems that have been invaded by alien species may not have the natural predators and competitors present in its native environment that would normally control their populations. Native ecosystems that have undergone human-induced disturbance are often more prone to alien invasions because there is less competition from native species.

Acid rain becomes more acidic when the pH decreases.

Page 129: Final Review LCHS 2013.

Which of the following is NOT a property of water?A. Water has the ability to filter out UV lightB. Water is a solvent, allowing many chemicals to dissolve in itC. When water freezes into ice, it expandsD. Water can change its temperature very quicklyE. Water has the ability to filter out red light

Page 130: Final Review LCHS 2013.

60 Answer: DWater has a very high specific heat capacity

as well as a high heat of vaporization both of which are a result of the extensive hydrogen bonding between its molecules. These two unusual properties allow water to moderate Earth's climate by buffering or resisting large fluctuations in temperature. The oceans absorb one thousand times more heat than the atmosphere and hold 80 to 90% of the heat due to global warming.

Page 131: Final Review LCHS 2013.

According to the graph, the increase in CO2 concentration, in parts per million, between 1970 and 1990 is closest to(A) 5 ppm(B) 30 ppm(C) 50 ppm(D) 340 ppm(E) 355 ppm

Page 132: Final Review LCHS 2013.

Which of the following most directly explains the periodic fluctuations of the curve?(A) Daily variations in air temperature(B) Daily variations in sea level(C) Seasonal variations in photosynthetic activity(D) Seasonal variations in ocean water temperature(E) Seasonal variations in human industrial activity

Page 133: Final Review LCHS 2013.

Which of the following would be considered a form of low-quality energy?

A. natural gasB. wind powerC. gasolineD. nuclear fissionE. coal

Page 134: Final Review LCHS 2013.

61 Answer: BEnergy quality can be thought of as

energy density. The less energy that is contained in a given volume, the lower its energy quality. When fossil fuels were created millions of years ago, solar energy was converted into chemical energy and then concentrated into small volumes. This gives these fuels a high energy density, thus high-quality energy.

Page 135: Final Review LCHS 2013.

Which of the following is NOT an anthropogenic source of atmospheric sulfur?

A. oil refiningB. using gasoline and diesel fuel C. smelters to produce a metal from its oreD. decay of organic materialE. incineration of municipal wastes

Page 136: Final Review LCHS 2013.

62 Answer: DThe term "anthropogenic" designates an effect or object resulting from human activity. Most of the sulfur on Earth is stored in oceans,

sedimentary rocks, and air particles (during evaporation). A very small percentage reaches the surface and exchanges with atmosphere.Sulfur is the 13th most abundant element in the Earth’s crust (0.1%) and the 9th most abundant in sediments. Sulfur content of rocks varies

considerably; for example, sedimentary rocks have about 0.38%, while igneous rocks have only 0.032%. Sulfur in the lithosphere is mobilized by the slow weathering of rock material. Dissolved in runoff, it moves with river water and is deposited in continental shield sediments in oceans. Eventually on the geological timescale, this uplifts to the surface again thus completing the geological part of the sulfur cycle.

Several sulfur compounds listed below are released into the atmosphere due to interaction of processes between the Earth’s surface and the atmosphere.

1. Carbonyl sulfide (COS): Carbonyl sulfide is the most abundant sulfur species in the atmosphere and is mainly produced by the decomposition processes in soil, in marshes and wetlands, and along ocean coasts and areas of ocean upwelling that are rich in nutrients. Oceans may act both as the source and the sink of COS. About 80% of total atmospheric sulfur is COS, but it is relatively inert and does not add much to the atmospheric sulfur pollution.

2. Carbon disulfide (CS2): Carbon disulfide is far more reactive than COS and has similar sources as carbonyl sulfide. It has a lifetime of 12 days only and its major sink is photochemical reactions. The most important source of the compound is microbial processes in warm tropical soils. Major secondary sources are marshes and wetlands along sea coasts with small anthropogenic inputs from fossil fuel combustion.

3. Dimethyl sulfide (DMS): Dimethyl sulfide is released from oceans in much greater amounts than COS or CS2 and is very rapidly oxidized to sulfur dioxide (SO2). Its concentrations are high during night, particularly in areas under some influence from continental sources.

4. Hydrogen sulfide (H2S): Hydrogen sulfide is produced in nature during anaerobic decay in soils, wetlands, salt marshes, and other areas of stagnant water with maximum concentrations occurring in tropical forests. Its highest concentrations occur at night and in early morning when photochemical activity is at a minimum.

5. Sulfur dioxide (SO2): Sulfur dioxide is released during the oxidation of H2S, which is emitted during the combustion of fossil fuels. In some industrialized areas such as eastern North America, more than 90% of SO2 is from anthropogenic sources. About 50% of SO2 is removed from the atmosphere by wet and dry deposition.

6. Sulfate aerosols (SO42−): Sulfate aerosol particles originate from sea spray and are one of the largest natural sources of sulfur to the

atmosphere. Most of the salt spray sulfate falls back to oceans, but some is carried over the continents to be included in other deposition processes.

Page 137: Final Review LCHS 2013.

Which of the following are characteristics of organisms that reproduce asexually?

I. They tend to grow in number exponentially.II. They rely on mutation in their DNA for variation(s).III. The large amount of variation in their genes makes them less susceptible to disease.

A. I onlyB. II onlyC. III onlyD. I and IIE. I, II, and III

Page 138: Final Review LCHS 2013.

63 Answer: DThe known methods of reproduction are broadly grouped into two main types: sexual and

asexual. In asexual reproduction, an individual can reproduce without involvement with another individual of that species (e.g., division of a bacterial cell into two daughter cells). Sexual reproduction typically requires the involvement of two individuals or gametes, one from each sex.

Advantages of asexual reproductive strategies include:1. Large numbers of offspring are reproduced very quickly from only one parent when

conditions are favorable.2. Large colonies can form that can outcomplete other organisms for nutrients and water.3. Large number of organisms mean that species may survive when conditions or the

number of predators change.4. Energy is not required to find a mate

Disadvantages of asexual reproductive strategies include:1. Offspring are genetic clones. A negative mutation can make asexually produced

organisms susceptible to disease and can destroy large numbers of offspring.2. Some methods of asexual reproduction produce offspring that are close together and

complete for food and space.3. Unfavorable conditions (i.e., extreme temperatures) can wipe out entire colonies.

Page 139: Final Review LCHS 2013.

Which of the following are environmental consequences of overfishing?

I. Resource depletionII. Increased biological growth ratesIII. Lower biomass levels

A. I onlyB. II onlyC. III onlyD. I and IIIE. I, II, and III

Page 140: Final Review LCHS 2013.

64 Answer: DOverfishing is the act whereby fish stocks are depleted to unacceptable levels, regardless

of water body size. Resource depletion, low biological growth rates, and critical low biomass levels result from overfishing. The ability of a fishery to recover from overfishing depends on whether the ecosystem's conditions are suitable for the recovery.

Dramatic changes in species composition can result in an ecosystem shift. For example, once trout have been overfished, carp might take over in a way that makes it impossible for the trout to reestablish a breeding population.

 There are three forms of overfishing:1. Growth overfishing occurs when fish are harvested at an average size that is smaller

than the size that would produce the maximum yield, which makes the total yield less than it would be if the fish were allowed to grow to an appropriate size. It can be countered by reducing fishing mortality to lower levels and increasing the average size of harvested fish to a size that will allow maximum yield.

2. Recruitment overfishing occurs when the mature adult population is depleted to a level where it no longer has the reproductive capacity to replenish itself. This can be mitigated by placing moratoriums, quotas, and minimum-size limits on a fish population.

3. Ecosystem overfishing occurs when the balance of the ecosystem is altered by overfishing. With declines in the abundance of large predatory species, the abundance of small forage type increases causing a shift in the balance of the ecosystem toward smaller fish species.

Page 141: Final Review LCHS 2013.

Sulfur dioxide in the presence of suspended air particulates produces human health effects often greater than can be accounted for by exposure to either pollutant alone. This effect is known as

A. bioaccumulationB. biomagnificationC. cross contaminationD. synergismE. trophic magnification

Page 142: Final Review LCHS 2013.

65 Answer: DOne of the best known sulfur-related pollution episodes took

place in London December 5–9, 1952, when high concentrations of air pollution were trapped to the ground due to specific weather conditions at the time. Because of the cold winter, Londoners were burning more low-quality, high-sulfur coal to heat their homes. The combination of sulfur oxides along with particulates and other air pollutants resulted in more than 4,000 deaths and another 8,000 in the months that followed; far more deaths than can be accounted for by exposure to any of the pollutants alone. Many deaths were due to respiratory tract infections from hypoxia (low levels of oxygen in the blood) and due to the obstruction of air passages from lung infections resulting from the SO2 being exposed to and trapped in lung tissues by the polluting air particles.

Page 143: Final Review LCHS 2013.

How much time will be required for a sample of tritium 3H to lose 75% of its radioactivity? The half-life of tritium is 12.26 years.

A. 3.07B. 6.13C. 12.26D. 24.52E. 36.78

Page 144: Final Review LCHS 2013.

66 Answer: DIf 75% is lost, then 25% (or 0.25) remains. (1/2)n = 0.25 n = 212.26 × 2 = 24.52 years An alternative way to solving the problem is(1/2)n = 0.25n log 0.5 = log 0.25n = log 0.25 / log 0.5n = 2 Tritium, 3H, is a radioactive isotope of hydrogen. The nucleus of tritium contains one proton and two

neutrons, whereas the nucleus of normal hydrogen 1H, contains one proton and no neutrons. Naturally occurring tritium is extremely rare on Earth, where trace amounts are formed by the interaction of the atmosphere with cosmic rays.

 The emitted electrons from the radioactive decay of small amounts of tritium cause phosphors to glow so

as to make self-powered lighting devices, which are now used in firearm night sights, watches, exit signs, map lights, and a variety of other devices and takes the place of radium, which can cause bone cancer. Tritium is also an important component in nuclear weapons and is used to enhance the efficiency and yield of fission bombs.

 Tritium is also an important fuel for research in controlled nuclear fusion.

Page 145: Final Review LCHS 2013.

Which of the following is most likely to be the direct result of lack of genetic diversity in a food crop such as corn?(A) Decreased kernel size(B) Decreased potential yield(C) Decreased dependence on chemical

fertilizers(D) Increased susceptibility to plant disease(E) Increased resistance to pests

Page 146: Final Review LCHS 2013.

Global warming can cause all of the following EXCEPT

A. a rise of sea levelsB. the lowering of the pH of the oceansC. increased temperature stratification in the oceansD. increased volcanic activityE. increased ozone destruction

Page 147: Final Review LCHS 2013.

67Answer: ETwo main factors have contributed to observed sea level rise. The first is thermal expansion: as ocean

water warms, it expands. The second is from the contribution of land-based ice due to increased melting.

As carbon dioxide dissolves in sea water, carbonic acid is formed, which has the effect of acidifying the ocean (lowering the pH). Future ocean acidification and climate change would impair a wide range of marine organisms and the effects rippling through the marine food webs.

Global warming will increase the temperature of surface waters first. Ocean water circulation patterns have far-reaching effects on global climate. Convective mixing is a dominant process due to thermal stratification of the water column. At low latitudes, warm, low-density surface waters float over a mass of much colder, high-density subsurface water. As warm surface water travels north, the temperature difference between surface and subsurface is diminished. Nutrient-depleted surface water cools and sinks, forcing deep water to rise. As deep water rises to the ocean surface, it brings a fresh pulse of nutrients that causes enhanced ocean productivity near the poles.

The retreat of glaciers and ice caps has the potential of increasing volcanism. Reduction in ice cover reduces the confining pressure exerted on the volcano, increasing stresses, and potentially causing the volcano to erupt. This reduction of pressure can also cause decompression melting of material in the mantle, resulting in the generation of more magma. The effects of current sea level rise could also include increased crustal stress at the base of coastal volcanoes from a rise in the volcano's water table and the associated saltwater intrusion, while the mass from extra water could activate dormant seismic faults around volcanoes. In addition, the wide-scale glacier melting is likely to slightly alter the Earth's rotational period and may shift its axial tilt creating further crustal stress changes.

Global warming and ozone depletion are independent phenomenon

Page 148: Final Review LCHS 2013.

Which Act established a system for controlling hazardous wastes from the time it was generated until its ultimate disposal (i.e., "cradle to grave")?

A. Comprehensive Environmental Response, Compensation, and Liability Act (Superfund)B. National Environmental Policy Act (NEPA)C. Pollution Prevention Act (PPA)D. Resource Conservation and Recovery Act (RCRA)E. Toxic Substances Control Act (TOSCA)

Page 149: Final Review LCHS 2013.

68 Answer DThe Resource Conservation and Recovery Act

(RCRA), enacted in 1976, is the principal federal law in the United States governing the disposal of solid waste and hazardous waste. Congress enacted RCRA to address the increasing problems the nation faced from its growing volume of municipal and industrial waste. RCRA set national goals for

Protecting human health and the natural environment from the potential hazards of waste disposal.

Reducing the amount of waste generated, through source reduction and recycling

Ensuring the management of waste in an environmentally sound manner.

Page 150: Final Review LCHS 2013.

Which Act tracks industrial chemicals currently produced in or imported to the United States?A. Comprehensive Environmental Response, Compensation, and Liability Act (Superfund)B. National Environmental Policy Act (NEPA)C. Pollution Prevention Act (PPA)D. Resource Conservation and Recovery Act (RCRA)E. Toxic Substances Control Act (TOSCA)

Page 151: Final Review LCHS 2013.

69 Answer: EThe Toxic Substances Control Act (TOSCA) passed by the

United States Congress in 1976 regulates the introduction of new or already existing chemicals.

TOSCA does not separate chemicals into categories of toxic and non-toxic but instead prohibits the manufacture or importation of chemicals that are not on its list. Manufacturers must submit premanufacturing notification to the U.S. Environmental Protection Agency (EPA) prior to manufacturing or importing new chemicals for commercial purposes. New chemical notifications are reviewed by the EPA, and if it finds an "unreasonable risk to human health or the environment," it may limit its use(s) or production volume to outright banning them.

Page 152: Final Review LCHS 2013.

Which Act established a trust fund to provide for cleanup when a responsible party or company could not be identified?

A. Comprehensive Environmental Response, Compensation, and Liability Act (Superfund)

B. National Environmental Policy Act (NEPA)

C. Pollution Prevention Act (PPA)D. Resource Conservation and Recovery Act

(RCRA)E. Toxic Substances Control Act (TOSCA)

Page 153: Final Review LCHS 2013.

70 Answer: ASuperfund is the common name for the

Comprehensive Environmental Response, Compensation, and Liability Act of 1980 (CERCLA), a federal law designed to clean up sites contaminated with hazardous substances. Superfund provides broad federal authority to clean up releases or threatened releases of hazardous substances that may endanger public health or the environment. The law authorized the Environmental Protection Agency (EPA) to identify parties responsible for contamination of sites and compel the parties to clean up the sites. Where responsible parties cannot be found, the Agency is authorized to clean up sites itself, using a special trust fund.

Page 154: Final Review LCHS 2013.

A U-shaped body of water formed when a wide meander from the main stem of a river is cut off over time as erosion and deposits of soil change the river's course is known as a(n)

A. eutrophic lackeB. oxbow lakeC. oligotrophic lakeD. mesotrophic lakeE. crater lake

Page 155: Final Review LCHS 2013.

71 Answer: BAs a mature river begins to curve, it cuts and

erodes into the outside of the curve and deposits sediment on the inside of the curve since the stream moves more rapidly on the outside of the curve and more slowly on the inside of the curve. Thus, as the erosion and deposition continues, the curve becomes larger and more circular. Eventually the river begins to cut the loop off by eroding the neck of the loop and the river breaks through at a cutoff and forms a new streambed. Sediment is then deposited on the loop side of the stream, cutting off the loop from the stream entirely.

Page 156: Final Review LCHS 2013.

Elements that cycle in the environment and that also have a gaseous phase at some point in their cycle include which of the following? I. Carbon II. Phosphorus III. Sulfur

(A) I only(B) III only(C) I and II only(D) I and III only(E) I, II, and III

Page 157: Final Review LCHS 2013.

Which of the examples listed below would be a positive externality?

A. Air pollution from fossil fuelsB. Industrial farm animal productionC. The cost of storing nuclear wasteD. Renewable energyE. Widespread use of antibiotics

Page 158: Final Review LCHS 2013.

72 Answer: DA negative externality (also called "external cost") is an action of a product on consumers that

imposes a negative side effect on a third party; it is "social cost." Many negative externalities are related to the environmental consequences of production and use.

Air pollution from burning fossil fuels causes damages to crops, (historic) buildings, and public health.

Industrial farm animal production, on the rise in the 20th century, resulted in farms that were easier to run, with fewer and often less-skilled employees, and a greater output of uniform animal products. However, the externalities with these farms include contributing to the increase in the pool of antibiotic-resistant bacteria because of the overuse of antibiotics; air quality problems; the contamination of rivers, streams, and coastal waters with concentrated animal waste; and animal welfare problems, mainly as a result of the extremely close quarters in which the animals are housed.

The cost of storing nuclear waste from nuclear plants for more than 1,000 years (over 100,000 for some types of nuclear waste) is included in the cost of the electricity the plant produces in the form of a fee paid to the government and the long-term risks of disposal of chemicals, which may remain permanently hazardous.

Antibiotic use contributes to antibiotic resistance, reducing the future effectiveness of antibiotics.

Renewable energy may create positive externalities as it reduces net environmental pollution.

Page 159: Final Review LCHS 2013.

Which of the following is least likely to be affected by global warming?

I. K-strategistsII. r-strategistsIII. Keystone species

A. I onlyB. II onlyC. III onlyD. II and IIIE. I, II, and III

Page 160: Final Review LCHS 2013.

73 Answer: BIn unstable or changing environments, r-selection predominates as

the ability to reproduce quickly is crucial. Traits that are characteristic of r-selection include high fecundity, small body size, early maturity onset, short generation time, and the ability to disperse offspring widely. Organisms who exhibit r-selected traits include bacteria, insects, weeds, and small rodents.

A keystone species is a species that has a disproportionately large effect on its environment relative to its abundance. Such species play a critical role in maintaining the structure of an ecological community, affecting many other organisms in an ecosystem and helping to determine the types and numbers of various other species in the community. Since keystone species can be either K-strategists (e.g., sea otters) or r-strategists (e.g., the herbivorous milfoil weevil insect, Euhrychiopsis lecontei); no correlation can be drawn as to global warming and its effect on keystone species.

Page 161: Final Review LCHS 2013.

Which of the following treaties, Acts, or Protocols addressed international trade in endangered species?

A. Montreal ProtocolB. Kyoto ProtocolC. London ProtocolD. CITESE. Endangered Species Act

Page 162: Final Review LCHS 2013.

74 Answer: DCITES (Convention on International Trade in

Endangered Species of Wild Fauna and Flora) is a multilateral treaty adopted in 1963 at a meeting of members of the International Union for Conservation of Nature (IUCN) and entered into force in 1975. Its aim is to ensure that international trade in specimens of wild animals and plants does not threaten the survival of the species in the wild, and it accords varying degrees of protection to more than 5,000 species of animals and 29,000 species of plants.

The Endangered Species Act is a law of the United States and does not regulate international trade in endangered species.

Page 163: Final Review LCHS 2013.

Which of the following events occur during an El Niño?

I. Oceanic surface temperatures along the North and South American coasts become cooler.

II. Upwellings of cold, nutrient-rich water are suppressed.

III. Prevailing easterly winds weaken.

A. I onlyB. II onlyC. III onlyD. I and IIE. I, II, and III

Page 164: Final Review LCHS 2013.

75 Answer: BOne of the oceanic processes altered during an El Niño year is upwelling, or the rising of

deeper colder water to shallower depths off the western coast of Peru—one of the world’s richest fisheries. Because of the frictional stresses that exist between ocean layers, surface water is transported at a 90 degree angle to the left of the winds in the Southern Hemisphere. Winds blowing northward and parallel to the coastline of Peru located in the Southern Hemisphere "drag" oceanic surface water away from the shore. As a result, nutrient-rich water rises from deeper levels to replace the surface water that has drifted away. It is these nutrients that are responsible for supporting the large fish population commonly found in the coastal areas off Peru.

However, the effectiveness of upwelling and its ability to support abundant marine life is greatly dependent upon the depth of the thermocline, which is the transition layer between the mixed layer at the surface and the deep water layer. The boundaries of these layers are based on water temperature.

The mixed layer is near the surface where the temperature is roughly that of surface water. In the thermocline, the temperature decreases rapidly from the mixed layer temperature to the much colder deep water temperature.

The mixed layer and the deep water layer are relatively uniform in temperature, while the thermocline represents the transition zone between the two.

However, a deeper thermocline (often observed during El Niño years) limits the amount of nutrients brought to shallower depths by upwelling processes, which greatly reduces the year's fish crop.

Page 165: Final Review LCHS 2013.

Individuals of the same species occupying a given area at the same time is known as a

A. groupB. population C. communityD. clusterE. niche

Page 166: Final Review LCHS 2013.

76 Answer: BA population is a group of individuals of the

same species occupying a particular geographic area. Populations may be relatively small and closed, as on an island or in a valley, or they may be more diffuse and without a clear boundary between them and a neighboring population of the same species. For species that reproduce sexually, the members of a population may interbreed either exclusively with members of their own population or with members of other populations.

Page 167: Final Review LCHS 2013.

Which of the following examples listed below is NOT an ecosystem service?

A. Detoxifies pollutantsB. Performs pest and disease controlC. Provides food (including seafood and game), crops, wild foods, and spicesD. Manages carbon sequestration and climate regulationE. All are examples of ecosystem services

Page 168: Final Review LCHS 2013.

77 Answer: EHumankind benefits from a multitude of

resources and processes that are supplied by natural ecosystems. Collectively, these benefits are known as ecosystem services and include products like clean drinking water and processes such as the decomposition of wastes.

Ecosystem services can be separated into four broad categories: (1) provisioning, such as the production of food and water; (2) regulating, such as the control of climate and disease; (3) supporting, such as nutrient cycles and crop pollination; and (4) cultural, such as spiritual and recreational benefits.

Page 169: Final Review LCHS 2013.

Which of the following statements is in accord with The Tragedy of the Commons?

I. Larger "commons" result in more personal responsibility.

II. Smaller "commons" result in greater personal responsibility.

III. The size of the "commons" has nothing to do with personal responsibility.A. I onlyB. II onlyC. III onlyD. I and IIIE. I, II, and III

Page 170: Final Review LCHS 2013.

78 Answer: BThe Tragedy of the Commons was written by ecologist Garrett Hardin and first

published in the journal Science in 1968.The tragedy of the commons is a dilemma arising from the situation in which multiple individuals, acting independently and rationally consulting their own self-interest, will ultimately deplete a shared limited resource, even when it is clear that it is not in anyone's long-term interest for this to happen.

The Tragedy of the Commons involves medieval land tenure in Europe, focusing on herders who share a common parcel of land on which they are each entitled to let their cows graze. It is in each herder's interest to put the next (and succeeding) cows he acquires onto the land, even if the quality of the common is damaged for everyone through overgrazing. The herder receives all of the benefits from an additional cow, while the damage to the common is shared by the entire group. If all herders make this individually rational economic decision, the common will be depleted or even destroyed, to the detriment of all. Hardin also cites modern examples, including the overfishing of the world's oceans and ranchers who graze their cattle on government lands.

Page 171: Final Review LCHS 2013.

Most of the world’s proven coal deposits are found in

A. ChinaB. RussiaC. United StatesD. CanadaE. Ukraine

Page 172: Final Review LCHS 2013.

79 Answer: CEstimates show that there are about 1,000 billion tons of

proven and recoverable coal reserves (~ 4,000 billion barrels of oil equivalent) left on Earth, which would last about another 150 years. Projections predict that global peak coal production may occur sometime around 2025 at 30% above current production, depending on future coal production rates.

Of the three fossil fuels (coal, oil, and natural gas), coal has the most widely distributed reserves; coal is mined in over 100 countries, and on all continents except Antarctica. The largest reserves are found in the United States, Russia, and China.

Country Total Percent of World Total

United State 237,295 22.6%

Russia 157.010 14.4%

China 114,500 12.6%

Page 173: Final Review LCHS 2013.

The Environmental Protection Agency has responsibilities in all of the following areas EXCEPT

A. resource conservationB. water pollutionC. waste disposalD. air pollutionE. noise pollution

Page 174: Final Review LCHS 2013.

80 Answer: AThe U.S. Environmental Protection Agency (EPA), an agency of the

U.S. federal government, was created for the purpose of protecting human health and the environment by writing and enforcing regulations based on laws passed by Congress.

Resource conservation falls under the jurisdiction of the National Resource Conservation Service (NRCS), which was originally established by Congress in 1935 as the Soil Conservation Service. NRCS has expanded to become a conservation leader for all natural resources, ensuring private lands are conserved, restored, and more resilient to environmental challenges (i.e., climate change).

Seventy percent of all land in the United States is owned privately. The NRCS works with private landowners through conservation planning and assistance designed to benefit the soil, water, air, plants, and animals and so effect productive lands and healthy ecosystems.

Page 175: Final Review LCHS 2013.

The land on a 100-acre farm is equally suited for grazing cattle and growing com. Of the following ways of distributing land use, which would produce the greatest number of calories for human consumption?

Acres for Acres forGrazing Cattle Growing Corn

(A) 100 0(B) 80 20(C) 50 50(D) 20 80(E) 0 100

Page 176: Final Review LCHS 2013.

Which of the following substances listed is NOT a teratogen?

A. smogB. benzeneC. drinking alcoholD. tobaccoE. X-rays 

Page 177: Final Review LCHS 2013.

81 Answer ATeratogens are substances or environmental agents that cause the

development of abnormal cell masses during fetal growth, resulting in physical defects in the fetus. The World Health Organization estimates that about 50% of all fertilized ova are lost within the first three weeks of development, that 15% of all clinically recognizable pregnancies end in spontaneous abortions, and that birth defects are known to occur in 3 to 6% of all newborns. They are the leading cause of infant mortality in the United States, accounting for more than 20% of all infant deaths. Seven to ten percent of all children will require extensive medical care to diagnose or treat a birth defect. It is extremely difficult to make accurate estimates of exposure risk due to the large number of pharmaceutical, industrial, and agricultural chemicals, which increase the risk of exposure to multiple agents and their potential synergistic effects, and human genetic heterogeneity, which contributes greatly to the individual level of threshold susceptibility. Smog has not been shown to be a teratogen.

Page 178: Final Review LCHS 2013.

The largest percent of domestic water use in the United States is used for

A. bathing and showeringB. watering lawns and gardensC. toiletsD. cooking and drinkingE. agriculture

Page 179: Final Review LCHS 2013.

82  Answer: BDomestic use includes water used around

the home. Up to 50% of household water goes toward outdoor use, and an estimated 50% of that is lost due to evaporation or landscape runoff due to overwatering.

Page 180: Final Review LCHS 2013.

An APES class is doing soil studies and determines that a sample of soil contains 30% clay, 20% silt, and 50% sand. Using the soil texture chart above, the soil would be classified as

A. silt, clay loamB. sandy, clay loamC. loamy sandD. silt loamE. sandy loam

Page 181: Final Review LCHS 2013.

83 Answer: B

Page 182: Final Review LCHS 2013.

The population in 1970 of a small mid-Western town was 35,000. The birth rate was measured at 20 per 1,000 population per year, while the death rate was measured at 5 per 1,000 population per year. People moving into the town were measured at 350 per year while those leaving the town were measured at 100 per year. By how much did the population increase (or decrease) in 1970?

A. increased by 300B. decreased by 775C. increased by 775D. increased by 1,200E. increased by 1,200

Page 183: Final Review LCHS 2013.

84 Answer: C Population Change = (Crude Birth Rate + Immigration) –

(Crude Death Rate) + Emigration

= [20(35) + 350] – [5(35)] + 100

= +775

Page 184: Final Review LCHS 2013.

Which of the following types of public lands would mining of natural mineral resources NOT be allowed on?

I. National ForestsII. National Resource LandsIII. National Wilderness Preservation System

A. I onlyB. II onlyC. III onlyD. I and IIE. I, II, and III

Page 185: Final Review LCHS 2013.

85 Answer: CMultiple Use Lands1. National Forest System: managed by the U.S. Forest Service; used for

logging, mining, livestock grazing, farming, oil and gas, recreation, sport hunting, and wildlife resources.

2. National Resource Lands: managed by the Bureau of Land Management; used for supply of domestic energy, minerals, and livestock grazing (with permit).

 Moderately Restricted Use Lands3. National Wildlife Refuges: managed by the U.S. Fish and Wildlife

Service; used for protecting habitats, breeding grounds, sport hunting, fishing, oil and gas extraction, mining, logging, grazing, and farming.

 Restricted Land Use4. National Park System: managed by the National Park Service; used for

camping, hiking, sport fishing, and boating.5. National Wilderness Preservation System: managed by National Park

Service, Forest Service, U.S. Fish and Wildlife Service, and the Bureau of Land Management; used for recreational activities only.

Page 186: Final Review LCHS 2013.

Which of the following is NOT a primary air pollutant from burning coal?

A. Carbon monoxideB. Particulate matterC. OzoneD. Sulfur dioxideE. Nitrogen oxides

Page 187: Final Review LCHS 2013.

86 Answer: CBurning coal generates 54% of the electricity generated in the United States and is the

single largest air polluter. Burning coal is a leading cause of smog, acid rain, global warming, and air toxics. In an average year, a typical coal plant generates:

• 3,700,000 tons of carbon dioxide (CO2), the primary human cause of global warming—as much carbon dioxide as cutting down 161 million trees.

• 10,000 tons of sulfur dioxide (SO2), which causes acid rain that damages forests, lakes, and buildings and forms small airborne particles that can penetrate deep into lungs.

• 500 tons of particulate matter, which can cause chronic bronchitis, aggravated asthma, and premature death.

• 10,200 tons of nitrogen oxide (NOx), which leads to the formation of ozone. Ozone inflames the lungs, burning through lung tissue making people more susceptible to respiratory illness. However, burning coal does NOT directly release significant amounts of ozone.

• 720 tons of carbon monoxide (CO), which causes headaches and places additional stress on people with heart disease.

• 220 tons of hydrocarbons, volatile organic compounds (VOC), which form ozone.• 170 pounds of mercury, which is so toxic that just 1/70th of a teaspoon (about the size

of a drop) of mercury deposited in a 25-acre lake will make the fish unsafe to eat.• 225 pounds of arsenic.• 114 pounds of lead, 4 pounds of cadmium, other toxic heavy metals, and trace

amounts of uranium.

Page 188: Final Review LCHS 2013.

Keystone species are species thatA. have the largest populations in a

communityB. occupy the highest niche in an ecosystemC. play a critical role in maintaining the

structure of an ecological communityD. form the base of the food pyramid in an

ecosystemE. are the most sensitive to and affected by

environmental degradation

Page 189: Final Review LCHS 2013.

87 Answer: CA keystone species is a species that has a disproportionately large effect on its

environment relative to its abundance. Such species play a critical role in maintaining the structure of an ecological community, affecting many other organisms in an ecosystem and helping to determine the types and numbers of various other species in the community. The role that a keystone species plays in its ecosystem is analogous to the role of a keystone in an arch. While the keystone is under the least pressure of any of the stones in an arch, the arch still collapses without it. Similarly, an ecosystem may experience a dramatic shift if a keystone species is removed, even though that species was a small part of the ecosystem by measures of biomass or productivity.

An example of a keystone species are sea otters, that live in the kelp forests off the coast of California. Sea otters are mammals who feed on a variety of marine invertebrates but who feed especially on sea urchins. Sea urchins are voracious herbivores that tend to feed on the base of the kelp, eating through the stipe until the whole plant detaches from the bottom and floats away. An overabundance of urchins can lead to overgrazing of the kelp forest. Once abundant in California, sea otters were hunted down for their pelts from a population of around 16,000 to near extinction. The resulting effect on the kelp forest was an increase in the barren areas because of the urchin populations. After they were placed under federal protection to increase their numbers and save them from extinction, sea otters were able to begin naturally controlling the urchins so that the kelp forests had a chance to recover.

Page 190: Final Review LCHS 2013.

A small, healthy lake had a dissolved oxygen (DO) content reading of 1 mg/L. A break in a major sewage line nearby spilled thousands of gallons of raw sewage into the lake. The DO content of the lake after the spill should

A. decrease as bacteria count increases and consumes DOB. decrease since sewage would provide nutrients for fish living in the lake and more fish would use more oxygenC. increase due to added amount of nutrients introduced into the lake, which would increase the rate of photosynthesis of aquatic plantsD. increase due to the added amount of oxygen and other gases released during the decay processE. should no change, as long as the sewage would remain dissolved

Page 191: Final Review LCHS 2013.

88 Answer: AEutrophication is the ecosystem response to the addition of artificial or natural

substances, such as nitrates and phosphates, through fertilizers or sewage, to an aquatic system.

 Following are ecological effects that can arise from eutrophication:• Decreased biodiversity• Changes in species composition and dominance• Toxicity effects• Increased biomass of phytoplankton• Toxic or inedible phytoplankton species• Increases in blooms of gelatinous zooplankton• Increased biomass of benthic and epiphytic algae• Changes in macrophyte species composition and biomass• Decreases in water transparency (increased turbidity)• Color, smell, and water treatment problems• Dissolved oxygen depletion• Increased incidences of fish kills• Loss of desirable fish species• Reductions in harvestable fish and shellfish• Decreases in perceived aesthetic value of the water body

Page 192: Final Review LCHS 2013.

Which of the following characteristics do NOT contribute to extinction?

A. Low reproductive rateB. Specialized feeding patternsC. Fixed migratory patternsD. Preys on livestock or peopleE. Offspring are small in size

Page 193: Final Review LCHS 2013.
Page 194: Final Review LCHS 2013.

Areas of low air pressure are typically characterized by __________ air and move toward areas where the air pressure is ____________. A. rising decreasingB. sinking, increasingC. rising, increasingD. sinking, decreasingE. sinking, not changing

Page 195: Final Review LCHS 2013.

90 Answer: AA low pressure system, or "low," is an area where the atmospheric pressure is

lower than that of the area surrounding it. Lows are usually associated with high winds, warm air, and atmospheric lifting. Because of this, lows normally produce clouds, precipitation, tropical storms, and cyclones. In addition, areas prone to low pressure do not have extreme diurnal (day vs. night) and seasonal temperatures because the clouds present over such areas reflect incoming solar radiation back into the atmosphere. As a result, under a low pressure system, the days are cooler and the nights are warmer.

Conversely, a high pressure system, or "high," is an area where the atmospheric pressure is greater than that of the surrounding area. High pressure areas are normally caused by a phenomenon called subsidence, meaning that as the air in the high cools it becomes denser and moves toward the ground. Pressure increases here because more air fills the space left from the low. Subsidence also evaporates most of the atmosphere's water vapor so high pressure systems are usually associated with clear skies and calm weather. Unlike areas of low pressure, the absence of clouds means that areas prone to high pressure experience extremes in diurnal and seasonal temperatures since there are no clouds to block incoming solar radiation or trap outgoing longwave radiation at night. Thus, such areas have higher high temperatures and lower low temperatures.

Page 196: Final Review LCHS 2013.

Examine the diagram below of a demographic transition model. Currently, the United States would be in what stage?

A. 1B. 2C. 3D. 4E. 5

Page 197: Final Review LCHS 2013.

91 Answer: DDemographic transition refers to the

transition from high birth and death rates to low birth and death rates as a country develops from a pre-industrial to an industrialized economic system. This is typically demonstrated through a demographic transition model.

Page 198: Final Review LCHS 2013.

The most rapidly growing renewable energy source since 1995 has been

A. hydropowerB. windC. solarD. geothermalE. nuclear

Page 199: Final Review LCHS 2013.

92 Answer: BRenewable energy currently accounts for about 7% of U.S.

energy consumption. Most of that comes from biomass and hydroelectric sources. Since 1995 the amount of energy produced by renewable sources has increased by 16%.

The most rapidly growing renewable energy source since 1995 has been wind power with the implementation of wind power increasing more than 2000%. Although this is very large growth, wind contributes less than 1% of the energy supply in the United States.

Solar has grown more than 55% since 1995 and the rapid fall in the per-kilowatt price of solar panels should support future growth. Geothermal has grown nearly 27%. New technologies and higher fossil fuel prices now make geothermal space heating projects cost competitive with fossil fuel units.

Page 200: Final Review LCHS 2013.

The release of methylmercury into the environment and the effect of its bioaccumulation occurred near this city.A. Bhopal, IndiaB. Chernobyl, UkraineC. Love Canal, New YorkD. Minimata, JapanE. Three Mile Island, Pennsylvania

Page 201: Final Review LCHS 2013.

93 Answer: DMinamata disease is a neurological syndrome caused by

severe mercury poisoning. Symptoms include ataxia, numbness in the hands and feet, general muscle weakness, narrowing of the field of vision, and damage to hearing and speech. In extreme cases, insanity, paralysis, coma, and death follow within weeks of the onset of symptoms. Minamata disease was first discovered in Minamata, Japan, in 1956 and was caused by the release of methylmercury in the industrial wastewater from the Chisso Corporation's chemical factory, which continued from 1932 to 1968. This highly toxic chemical bioaccumulated in shellfish and fish in Minamata Bay which when eaten by the local populace resulted in mercury poisoning, resulting in 2,265 victims, 1,784 of whom died.

Page 202: Final Review LCHS 2013.

The building of a residential community on land that contained 21,000 tons of buried toxic chemicals occurred in this city.A. Bhopal, IndiaB. Chernobyl, UkraineC. Love Canal, New YorkD. Minimata, JapanE. Three Mile Island, Pennsylvania

Page 203: Final Review LCHS 2013.

94 Answer: CLove Canal was a neighborhood in Niagara Falls, New

York. In the mid-1970s Love Canal became the subject of national and international attention after it was revealed that the site had formerly been used to bury 21,000 tons of toxic waste by Hooker Chemical Company.

The construction of the housing development combined with particularly heavy rainstorms over time released the chemical wastes into the environment. As a result of the buried chemicals, high rates of unexplained illnesses, nervous disorders, cancers, miscarriages, birth defects, and mental retardation were common in the area. A survey conducted by the Love Canal Homeowners Association found that 56% of the children born from 1974 to 1978 had at least one birth defect.

Page 204: Final Review LCHS 2013.

Approximately what percentage of the world’s population lives in developing countries?A. less than 5%B. 25%C. 50%D. 80%E. 95%

Page 205: Final Review LCHS 2013.

95 Answer: DThe development of a country is measured with such

factors as income per person, gross domestic or national product, life expectancy, the rate of literacy, and the availability of health care. Developing, also known as less- or least-developed countries, are in general countries that have not achieved a significant degree of industrialization relative to their populations and that have, in most cases, a medium to low standard of living; examples include Egypt, the Philippines, and Mexico. They include countries with a low living standard, undeveloped industrial base, and low Human Development Index (HDI) relative to other countries. There is also a strong correlation between low income and high population growth

Page 206: Final Review LCHS 2013.

Which of the following choices below shows the correct order for the stages of succession for a temperate deciduous forest?A. Perennial plants and grasses annual plants shrubs pines hardwood treesB. Annual plants and grasses perennial plants shrubs pines hardwood treesC. Annual plants perennial plants and grasses shrubs pines hardwood treesD. Perennial plants grasses annual plants shrubs pines hardwood trees E. Grasses annual plants perennial plants shrubs pines hardwood trees

Page 207: Final Review LCHS 2013.

96 Answer: CEcological succession is the process by which an ecological community undergoes orderly and

predictable changes following a disturbance or initial colonization of new habitat. Succession that begins in new habitats, not influenced by pre-existing communities is called primary succession, whereas succession that follows disruption of a pre-existing community is called secondary succession. Physical factors that determine the nature of the community that develops in an area are temperature, the amount of rainfall, soil type, and the amount of sunlight reaching the plants.

Temperate deciduous forests occur south of the taiga in the Northern Hemisphere. They grow in areas with a growing season between 140 and 300 days and with 30–60 inches (75–150 cm) rain/year.

They have a complex understory with some species (particularly smaller species) flowering in the spring before the canopy leafs out. For animals in a temperate forest, nuts (seeds of the dominant trees) are an important food source. Many of the food chains in the temperate deciduous forest, however, are detritus based. Many organisms depend on nutrients and energy released from the leaves that drop to the forest floor each fall (annual leaf fall before the cold season is a characteristic of the temperate deciduous forest). While the soil is rich, more of the nutrients are tied up in the trees, to be released when the trees die.

Pioneer species (annuals) that produce large quantities of seeds and that are disseminated by the wind are able to colonize large empty areas. They are capable of growing under direct sun exposition and are the first to inhabit vacant areas in the temperate deciduous forests. Once these plants have produced a closed canopy, the lack of direct sunlight at the soil level makes it difficult for their own seedlings to develop. It is then that the opportunity for more shade-tolerant species such as perennials and grasses can become established. When these more shade-tolerant plants die, the even more shade-tolerant shrubs and trees begin to replace them in kind. The shade-tolerant species are capable of growing under the canopy and, therefore, in the absence of catastrophes, will stay. At this point, the community has reached its climax. When another catastrophe arrives destroying the climax community, the opportunity for the pioneer species will be open once again.

Page 208: Final Review LCHS 2013.

The "heat island effect" is most associated with

A. desertB. islandsC. citiesD. rural areasE. stratospheric warming

Page 209: Final Review LCHS 2013.

97 Answer: CAn (urban) heat island is a metropolitan area, which is significantly warmer

than its surrounding rural areas. The temperature difference usually is larger at night than during the day and is most apparent when winds are weak. Seasonally, it is seen during both summer and winter. The main cause of the urban heat island is modification of the land surface by urban development, which uses materials that effectively retain heat. Waste heat generated by energy usage is a secondary contributor. As a population center grows, it tends to expand its area, and increase in its average temperature.

 Monthly rainfall is greater downwind of cities, partially due to the heat island

effect. Increases in heat within urban centers increases the length of growing seasons and decreases the occurrence of weak tornadoes. It also decreases air quality by increasing the production of pollutants such as ozone and decreases water quality as warmer waters flow into area streams.

 Mitigation of the urban heat island effect can be accomplished through the use

of light-colored roofs and the use of lighter-colored surfaces in urban areas, which reflect more sunlight and absorb less heat.

Page 210: Final Review LCHS 2013.

Which type of radiation does the stratospheric ozone layer entirely block?

A. UV-AB. UV-BC. UV-CD. InfraredE. Gamma

Page 211: Final Review LCHS 2013.

98 Answer: CThe stratospheric ozone layer is a layer in the Earth's atmosphere containing

about 10 parts ozone per million parts air, with the average ozone concentration in Earth's atmosphere being about 0.6 parts per million parts air. The ozone layer is mainly found in the lower portion of the stratosphere from approximately 12–19 miles (20–30 km) above the Earth, though the thickness varies seasonally and geographically. Although the concentration of the ozone in the ozone layer is very small, it is vitally important to life because it absorbs biologically harmful ultraviolet (UV) radiation coming from the sun. Extremely short UV radiation is divided into three categories based on its wavelength and are referred to as UV-A (400–315 nm), UV-B (315–280 nm), and UV-C (280–100 nm). UV-C, which would be very harmful to all living things, is entirely screened out by stratospheric ozone. Some UV-B radiation does penetrate the ozone layer and reaches the Earth’s surface. UV-B can be harmful to the skin and is the main cause of sunburn and skin cancer and excessive exposure can even cause genetic damage. The intensity of UV-B radiation at the top of the stratosphere is 350 million times stronger than at the Earth’s surface.

Ozone is transparent to most UV-A, so most of this longer wavelength, lower energy ultraviolet radiation does reach the Earth’s surface. UV-A is significantly less harmful to DNA than UV-B, although it may still potentially cause indirect genetic damage in skin.

Page 212: Final Review LCHS 2013.

Which method below tries to mimic a basic function of economic markets by setting an economic standard for measuring the success of environmental projects and/or programs?

A. External cost analysisB. Marginal cost analysisC. Cost-benefit analysisD. Sustainable cost analysisE. Risk analysis

Page 213: Final Review LCHS 2013.

99 Answer: CCost-benefit analysis provides an organizational framework for identifying, quantifying,

and comparing the costs and benefits (measured in dollars) of a proposed (environmental) policy action. The final decision is informed (though not necessarily determined) by a comparison of the total costs and benefits.

Cost-benefit analysis has two purposes:To determine if it is a sound investment/decision (justification/feasibility) andTo provide a basis for comparing projects. It involves comparing the total expected cost

of each option against the total expected benefits, to see whether the benefits outweigh the costs, and by how much.

The following steps that comprise a generic cost-benefit analysis:• List alternative projects/programs.• List stakeholders.• Select measurement(s) and measure all cost and benefits elements.• Predict outcome of cost and benefits over relevant time period.• Convert all costs and benefits into a common currency.• Calculate net present value of project options.• Adopt recommended choice.• Measure the effects on users or participants.• Measure externality effects.• Measure the social benefits.

Page 214: Final Review LCHS 2013.

During the December solstice, the sun is directly overhead at the

A. North PoleB. South PoleC. equatorD. Tropic of CancerE. Tropic of Capricorn

Page 215: Final Review LCHS 2013.

100 Answer: EThe December solstice occurs between December 20th and

December 23rd. It is at a time when the North Pole is tilted 23.5° away from the sun. On this date, all places above 66.5° north latitude (Arctic Polar Circle) are in darkness, while locations below latitude of 66.5° south (Antarctic Polar Circle) receive 24 hours of daylight. The sun is directly overhead on the Tropic of Capricorn in the Southern Hemisphere during the December solstice. It also marks the longest day of the year in terms of daylight hours for those living south of the equator.

For an observer in the Northern Hemisphere, the December solstice marks the day of the year with the least hours of daylight. Those living or traveling north of the Arctic Circle toward the North Pole will not be able to see the sun during this time of the year.